[Page 161]Abstract: A novel theory for the origins of Lehi’s vision of the Tree of Life has been offered by Rick Grunder, who argues that the story was inspired by a June 1829 visit to Rochester where Joseph could have seen a “great and spacious building,” a river, an iron railing, and even fruit trees. The purported source for the great and spacious building, the Reynolds Arcade, has even been suggested by one critic as a place where Joseph might have found “rare maps,” such as a map of Arabia that could have guided his fabrication of Lehi’s trail. As beautiful as such theories may be to their champions, they utterly fail to account for Nephi’s text.

Among the shortcomings of Grunder’s theory and creative extensions of it, the timing is problematic, for Joseph’s visit to Rochester likely occurred well after 1 Nephi was dictated. The proposed parallels offer little explanatory power for Book of Mormon creation. (For comparison, two online appendices for this article have been provided to illustrate how interesting random parallels can be found that may be more compelling than those Grunder offers.1) Further, any inspiration from a visit to Rochester as the plates of Nephi were being translated fails to account for the influence of Lehi’s vision and Nephi’s text on other portions of the Book of Mormon that were translated long before Joseph’s trip to Rochester. Finally, Nephi’s account of the vision of the Tree of Life and surrounding text cannot be reasonably explained by Grunder’s theory of last-minute fabrication inspired by Rochester or by any other theory of modern fabrication, as it is far too rooted in the ancient world and far too artfully crafted to have come from Joseph Smith and his environment.

Critics[Page 162] of the Book of Mormon have offered many theories over the years to explain the origins of the “voice from the dust,” the Book of Mormon. As many of its ridiculous blunders have gradually been eroded by ongoing scholarship to reveal surprising bedrock, often rich in gems, beneath its dusty surface, the old theories of Joseph as a bumbling fool have required updating. Efforts to ascribe Book of Mormon strengths to third parties like Solomon Spaulding or Sidney Rigdon have not proven fruitful.2 Increasingly critics point to modern parallels and sources of information that Joseph or his allies theoretically could have mined to make up various elements in the text, though this seems to require an ever growing library of source materials that, in reality, may not have been easy for Joseph to have accessed even if he knew where to look. This was one of the points of the recent two-part article in The Interpreter on the evidences for Lehi’s Trail, which examined the unsuitability of modern maps of Arabia to account for the impressive evidences for the authenticity of Nephi’s account of the trek through Arabia.3

A major portion of the two-part series on Lehi’s trail responded to the many arguments of an anonymous writer for Faith Promoting Rumor, RT, who has responded to the articles.4 RT’s displeasure with the many still unspecified methodological errors, rhetorical posturing, and “preposterous and uninformed” attitudes toward the scholarship of biblical minimalists are all outside the scope of this inquiry, though treated briefly elsewhere.5 Instead, here we focus on an intriguing new lead from Rick Grunder, advanced and enhanced by RT, regarding Book of Mormon origins.

A Beautiful Theory: Rochester as a Major Source for the Final (Initial) Chapters of the Book of Mormon

To his credit, RT’s response to the recent Interpreter articles on Lehi’s trail did somewhat acknowledge one point, namely, the low probability of Joseph having ever accessed one of the maps of Arabia that had the name Nehem or Nehhm. RT had argued that such maps must have served as the source of Nahom in 1 Nephi 16:34.

On the subject of maps, I agree with Lindsay about their rarity. In a strictly historical sense, the likelihood of JS encountering one in rural Western New York wasn’t very high. But my argument for dependence on a map doesn’t actually rise or fall on the question of accessibility, but on a combination of other factors, e.g. the BoM’s fictional character, the vague geography of the journey through Arabia vs. the precision of [Page 163]the location Nahom, the similarity between Irreantum and Erythraeum, other map features, etc. I assume that there were more maps available to JS in his world than we have record. Also, Rick Grunder has informed me that near to the time JS was dictating 1 Nephi he may have visited the Reynolds Arcade in Rochester, New York, which seems to form the material background for parts of the story of Lehi’s dream. At the time the Arcade was an exceptionally large and lavish building that featured a library, rare maps, and periodicals. [Here RT links to Grunder’s PDF file.6]7

This shows some progress, perhaps, compared to his previous essay on Nahom that approvingly quoted Philip Jenkins: “The map evidence makes it virtually certain that Smith encountered and appropriated such a reference, and added the name as local color in the Book of Mormon.”8 Now RT at least recognizes that access to a Nahom-related map may not have been so likely, but still seems persuaded that the Book of Mormon ultimately depends on a map through some unspecified means. The “virtual certainty” appears to still be in place, but perhaps with bigger questions marks over how it came to pass.

In the above passage from RT, I was surprised by his endorsement and implicit expansion of Rick Grunder’s theory regarding the Reynolds Arcade in Rochester, New York. Grunder maintains that this building was the inspiration for Lehi’s great and spacious building in the opening chapters of the Book of Mormon. Grunder is a master of finding creative parallels for Book of Mormon elements (see the review of his work by Ben McGuire9), and his theory that Rochester may have been Joseph’s inspiration for parts of 1 Nephi is perhaps the crowning achievement of his lifetime of looking for evidence of modern origins of the Book of Mormon, as Grunder suggests in a recent blog post:

Across my desk, over the decades, have passed many sobering relics: Martin Harris’ 1830 Book of Mormon, for instance, or another used at Hawn’s Mill. … Yet nothing carried more impetus for what I regard as my life’s work (though I couldn’t have imagined it at the time) than a little Rochester brochure purchased so casually from a catalog nearly thirty years ago [a pamphlet on the majestic Reynolds Arcade in Rochester].

Ironically, that minor piece isn’t particularly important now. It was merely an agent provocateur. But how provocative the results! I could only say it plainly in retrospect: “The most [Page 164]interesting Mormon parallel situations from the nineteenth century will offer similarities which are at least as good as ancient ones. And, they will be much more available and straightforward. So it is with the narrative of Rochester, New York.” When Joseph Smith was dictating the dream of the iron rod in 1829, he was also looking at a substantial iron railing hundreds of feet long. This is not some dodgy, speculative connection. Rather, it is an inescapable conjunction of history, in my long-studied opinion. That rod protected a narrow path from which one might fall easily to one’s death in the large, fast-flowing river below, on the other side of which rose — high into the air — a great and spacious building filled with the proudest, best-dressed people of Rochester, New York. This was the Reynolds Arcade, which according to historian Paul E. Johnson, “dramatized” the segregation of social classes when it opened in 1828 — one year before Joseph Smith and Martin Harris walked that same corner in search of a printer for the upcoming Book of Mormon then in preparation — at a moment in June 1829 that was close as I can calculate to the time when the dream of the iron rod was first spoken by Joseph Smith. …

Different perspectives inevitably divide Mormon Studies, and many of the entries I have written on other topics are intended merely as alternative views. In the case of the Reynolds Arcade, however, I see no way out. The first reviewer of Mormon Parallels observed that “the image of the brand-new Reynolds Arcade will convince even the most stout believers of Joseph Smith’s prophetic calling that the image of that structure could not but have been present in his mind.” [emphasis added]10

Is Grunder’s conclusion an “inescapable conjunction of history,” leaving Mormon believers with “no way out”? Or does Grunder’s crowning achievement bear a crown somewhat less regal than Grunder imagines?

The details of Grunder’s find are shared in a PDF file from his work, Mormon Parallels: A Bibliographic Source.11

Figure 1. The great and spacious Reynolds Arcade in Rochester. The observatory on top was missing at the time.

Figure 1. The great and spacious Reynolds Arcade in Rochester. The observatory on top was missing at the time.

[Page 165]According to Grunder, as Joseph neared the end of his translation work of the Book of Mormon in June 1829, near the beginning of the translation of the small plates of Nephi, he got the idea for the “great and spacious building” in Lehi’s dream when he made a trip to Rochester to look for a printer of the nearly completed manuscript. Inspired by a large building in Rochester, the Reynolds Arcade, towering at four-and-a-half stories, and just a block or so from an iron railing on an aqueduct that crossed the local Genesee River, Joseph thought of the iron rod and the “great and spacious building” that play such a significant role in 1 Nephi. Joseph then quickly added that material to his dictated translation and voila, 1 Nephi was written, followed by the rest of the small plates material in short order.

An 1827 map of Rochester (Figure 3) shows the relationship between the aqueduct, the nearby bridges, and the Reynolds Arcade. Caution must be used in considering images of the aqueduct, for the original aqueduct was torn down in 1840 and rebuilt with a different structure.12 A good collection of images (engravings and photos) of the aqueduct at various times is provided on ErieCanal.org.13 The aqueduct is no longer there and has been converted into a conventional bridge:

[Page 166]With the construction of the Barge Canal in 1918, the canal was moved south of the city of Rochester. Since the Genesee River Aqueduct was no longer needed, a road deck for Broad Street was built atop the aqueduct in 1922–1924, and the aqueduct was modified internally to carry the tracks of the Rochester Subway (or Rochester Industrial and Rapid Transit Railway) from 1927 to 1956. The Broad Street Bridge deck was rebuilt as it stands today in 1973–1974.14

Figure 2. Reynolds Arcade, from an 1844 sheet music publication.15

Figure 2. Reynolds Arcade, from an 1844 sheet music publication.15

An engraving from 1830 (Figure 4) illustrates what Joseph could have seen. An iron railing, a path, and a river are collocated, though not in the way the Book of Mormon describes. Could it have been viewed by Joseph just in the nick of time for translating 1 Nephi? Do these elements really offer such clear parallels as to be inescapable explanations for the details of Lehi’s dream?[Page 167]

Figure 3. Detail of an 1827 map by Elisha Johnson as marked by Grunder. The R is the location of the Reynolds Arcade; I added the label for the aqueduct.16

Figure 3. Detail of an 1827 map by Elisha Johnson as marked by Grunder. The R is the location of the Reynolds Arcade; I added the label for the aqueduct.16

Figure 4. 1830 engraving shows the Erie Canal aqueduct passing over the Genesee River in Rochester.17

Figure 4. 1830 engraving shows the Erie Canal aqueduct passing over the Genesee River in Rochester.17

How Clear Are the Parallels?

[Page 168]Grunder makes an interesting case. There was an iron barrier — a fence or guardrail — running along the impressive aqueduct of the Erie Canal that crosses the Genesee River in Rochester. This barrier protected workers towing boats across the aqueduct as they moved along the towpath next to the canal. The iron railing and the aqueduct were not far from the original Reynolds Arcade, built in 1828, which in Grunder’s view was the great and spacious building that inspired Joseph. It was a four-and-a-half story building with a unique open interior like modern malls. It had shops on the first and second floors, including a popular post office. While four stories may not seem tall enough to qualify as Lehi’s towering edifice that “stood as it were in the air, high above the earth” (1 Nephi 8:26), it did originally include a small but lofty structure on the top that went well above the four-story bulk of the building, extending as high as 90 feet.18 So if Joseph were the author of the Book of Mormon, he could have seen that building and been wowed.

The building, in fact, was associated with merchants and circles of wealth and power in Rochester, like many elegant buildings around the world, and so could be a fitting receptacle for the worldly mockers in Lehi’s dream. As Diane Shaw writes in a chapter describing the Rochester Arcade, “The rare building type distinguished Rochester, giving it a particular panache among all American cities. The Arcade embodied the aspirations of the merchant class, whose members sought an efficient, profitable, beautiful commercial experience whose very space, products, and image served their business and social needs.”19 So if Joseph needed inspiration for a great and spacious building, the Arcade could do. Then he could have wandered a block or two away to the Genesee River to see the aqueduct and its railing, thus acquiring the idea of an iron rod and a river. The river, an iron railing, a narrow towpath next to the railing, and a large, elegant building, all in the same town — these parallels appear notable. On top of that, the region around Rochester had fruit trees. With those elements all brought together, the origin of Lehi’s dream should be overwhelmingly clear, in Grunder’s view, and can’t be dismissed as utterly irrelevant to Lehi’s dream. But do they really provide a reasonable basis for suspecting modern origins for 1 Nephi?

First, note that the parallels are not quite as clear and direct as Grunder might wish for. The Rochester iron rod is on an aqueduct going across the Genesee river, not running along the bank of the river, as in Lehi’s dream (1 Nephi 8:19), nor does it lead to a tree and the head of a fountain (1 Nephi 8:20). The river does not divide the wicked in [Page 169]the great and spacious building from the rod of iron in Lehi’s dream (1 Nephi 8:26). The narrow towpath on the aqueduct does not lead to a tree adjacent a fountain of water, and it is unclear whether pedestrians could freely use it since it was obviously a structure for Erie Canal workers, apparently towers on horseback. While the iron fence would keep workers and horses from plummeting off the aqueduct into the river, pedestrians still would face the safety issue of having the exposed canal at their side, where they could fall into water or several feet down onto stone when the canal was dry, as apparently was the case in winter, making it seem unsuitable for ordinary traffic. Further, it would not be necessary for pedestrians since bridges for regular traffic were nearby, just a block away on both sides of the aqueduct, so the idea of numerous pedestrians pressing forward via the rod or wandering into strange paths adjacent the rod may not fit what Joseph could have seen.

Nevertheless, there was an iron railing and a river and not too far away was a rather tall building for upstate New York standards. While New York fruit trees might not have been very impressive or fruity in early June, still, Joseph would have recognized them as fruit trees. So Joseph could have seen all that in his 1829 trip to Rochester, where he tried to find a printer to print the Book of Mormon. Since the books of Nephi and the other material from the small plates were apparently translated in June 1829, offering a replacement for the material originally in the lost 116 pages, it would seem that an early June visit could have occurred before 1 Nephi was completed. If the visit were early enough, it would be theoretically possible for Joseph to have used the Reynolds Arcade and other elements from Rochester as inspiration for Lehi’s dream as he wrapped up the dictation for the Book of Mormon project. Grunder is ecstatic with this find.

RT is intrigued by the iron rod plus Reynolds Arcade theory, and offers what I feel is a clearer, more succinct summary of the strengths of Grunder’s position than Grunder does, with some of his own thoughts:

In my view, the strength of the parallel relates to the conjunction of a long rod of iron and narrow path, a large swift flowing river and nearby falls (“terrible gulf”), and an exceptionally large and lavish building nearby. The appearance of a rod of iron in this setting is particularly important, since it is clearly not an ancient motif. There were no rods of iron set next to rivers at the time of Nephi in the Old World. I have always wondered where the notion could have come from, and so its [Page 170]presence here in a setting highly evocative of Lehi’s dream and at a place JS is known to have visited is difficult to ignore.20

The link between the gulf mentioned in the Book of Mormon and the falls on the Genesee River depends upon Grunder’s find in an 1824 dictionary that “gulf” means “a bay; a whirlpool,” which he feels accurately describes the falls on the Genesee River, a couple of blocks downstream from the aqueduct.21 But Grunder’s theory does not need the falls; a strong river or any noteworthy body of water could be considered a gulf, if one is looking for parallels.

The alleged anachronism of a rod of iron in an ancient setting is addressed below.

Of course, the central image of Lehi’s dream is not the rod nor the building, but the tree of life, and here Grunder’s model does not provide a reasonable inspiration in Rochester apart from an 1838 article proclaiming that Rochester fruit markets showed the region was known for its fruit trees22 — an observation one can make for many other parts of the U.S.

Beyond the parallels to Lehi’s dream that Grunder and RT see in Rochester, RT sees even more in the Reynolds Arcade than Grunder did. RT, who has been looking toward rare European maps that might have been inspiration for the details of Lehi’s trail, hopefully notes that at the time of Joseph’s visit to Rochester, the Reynolds Arcade, according to Grunder, “featured a library, rare maps, and periodicals.”23 Could the Arcade not only provide inspiration for a major part of Nephi’s record, but also solve the mystery of the “Dream Map,” offering the source to the rare maps of Arabia that Joseph would need to complete the Book of Mormon? In a way, it’s a beautiful theory.

Below we’ll consider the components of this “beautiful theory,” beginning with the purported rare maps and then continuing with the various components of Lehi’s dream. Meanwhile, those interested in learning more about the Reynolds Arcade and its history and architecture (a great tidbit of American architecture, complete with a “Chinese pagoda” on top), there are a variety of further materials to consider.24

The Arcade: A Source of Rare Maps?

RT recognizes that the Arcade was a noted source of information in Rochester. But did the Arcade house “rare maps” that Joseph could have accessed and, perhaps, used to guide the route of Lehi’s trail? What is the evidence for this?

[Page 171]RT cites Grunder, who cites an 1830 source that mentions maps at the Athenaeum, an educational institute in Rochester which was housed in the Arcade. According to Grunder:

”Under its [the Arcade’s] roof,” reported New York City’s Monthly Repository magazine in 1830, “are six stores, an extensive boarding house, the post office, printing and exchange offices, the Atheneum [sic], justices’ and lawyers’ offices, &c. The Atheneum is very creditable to the place, having a very valuable library, maps, the periodicals and newspapers from various parts.”25

At least as of 1830, the Athenaeum housed maps. But where are the “rare maps” of RT? And more to the point, where are the rare maps of Arabia that might have inspired Joseph? No evidence that I have found supports that wishful notion.

To flesh out the theory, it helps to know a little more about the Athenaeum.

According to the Rochester Institute of Technology’s (RIT’s) “History of RIT,”26 the Athenaeum was founded in 1829 by Colonel Nathaniel Rochester and other Rochester community leaders “for the purpose of cultivating and promoting literature, science, and the arts.” It was housed in the Reynolds Arcade, and had a book collection that would grow over the years until 1847, when the Athenaeum merged with the Mechanics Literary Association, founded in 1836 by William A. Reynolds (son of Abelard Reynolds), to form the Rochester Athenaeum and Mechanics Association. The resulting merged library would have over 8,000 volumes, making it a truly significant library. It would be a major part of the roots of RIT. When it was in the Reynolds Arcade, it included a meeting room and a small reading room with a library, provided by Abelard Reynolds. Though small in 1829, could it have offered what Joseph needed?

One clue about the contents of the Athenaeum comes from the 1839 publication of the contents of the Rochester City Library, which included the contents of the Athenaeum.27 While there appears to be precious little in the way of maps listed there, there is a noteworthy ten-volume series of books on geography and world travel by Josiah Conder called Modern Traveler, which included a volume on Arabia that includes a fold-out map of Arabia.28 The series is listed as being printed in 1825, but apparently another edition came out in 1830 and another in 1833.29 Regardless of its print date, based on the catalog number, the acquisition of that volume by the Athenaeum appears to be well after 1830.30 Thus, [Page 172]there is little chance that Joseph could have seen the Arabia volume of The Modern Traveler series in the Athenaeum.

If Joseph had been able to access Conder’s volume on Arabia to study its map, what would he have gained? A limited resolution version of the map in black and white only is available online,31 and a small version of Conder’s full 1825 map is available online from Rooke Books of Bath, England,32 who own of a copy of Conder’s 1825 original. The owners of Rooke Books kindly provided a photograph of the region around Sana’a (Figure 5) to help me see if Nehem, Nihm, Nehm, or some other word related to ancient Nahom can be seen.33

Figure 5. A section of Josiah Conder’s 1825 map of Arabia from The Modern Traveler, courtesy of Rooke Books, Bath, England. The Nikkum Mountains are near the center, northeast of Sanaa.

Figure 5. A section of Josiah Conder’s 1825 map of Arabia from The Modern Traveler, courtesy of Rooke Books, Bath, England. The Nikkum Mountains are near the center, northeast of Sanaa.

There appears to be nothing that Joseph could have used to come up with the name Nahom. However, the Nikkum Mountains, present in the [Page 173]region associated with the Nihm tribe, may have a connection to Nahom that a student of Hebrew or Arabic might appreciate. As discussed in “Joseph and the Amazing Technicolor Dream Map: Part 1,”34 one of the objections made against LDS efforts to identify Nephi’s Nahom with the ancient and modern Nihm tribe is that the Hebrew word nacham (Strong’s H5162, נָחַם), a word associated with death, mourning, and comforting suggestive of a wordplay in Nephi’s text, has a hard H while the Arabic Nihm and the ancient South Arabian NHM of the Nihm tribe employ a softer H.35 But the existence of the Nikkum name, in the text from both Carsten Niebuhr36 and Conder37 and on Conder’s map, could be transliterated from a related NHM word with the hard H, which sometimes is transliterated with a K or KK. In other words, it could be that a local dialect in Yemen once used a hard H for an NHM name, possibly suggesting that previously a hard H may have been used. Thus, it may be that the linguistic gap between Arabic Nihm/Nehem/Nehhm/NHM and Nephi’s Nahom may not be as large as RT would make it. Perhaps Nephi heard locals naming the region with a Nikkum-like word that readily suggested the Hebrew cognate nacham. In any case, Hebrew has cognate NHM words with both soft and hard H (the soft-H word being naham, Strong’s H5098, נָהַם)38) casting further doubt on RT’s assertion that Nephi would not have recognized a relationship between the local NHM name and NHM in Hebrew.

Thus, the best candidate for a map of Arabia in Rochester’s Athenaeum appears to have been unavailable to Joseph in June or July of 1829, and even if he had seen it, it would not have given him any guidance to come up with the multiple impressive evidences supporting the authenticity of Lehi’s trail. Further, that map, while not offering guidance on the existence of Nahom, weakens the argument of critics that the local NHM name in Yemen would be unrecognizable as the Hebrew NHM word associated with death, mourning, and comfort due to a difference in the H sounds. Joseph, having not yet studied Hebrew as the Book of Mormon was being translated, would be unlikely to make a connection between Nikkum and Nahom, but such a connection may exist, especially in light of the Nikkum Mountains on Conder’s map being in essentially the right spot for the territory of the Nihm tribe and ancient Nahom.

Apart from the inability to explain the body of evidence related to Nahom, there is still no reasonable map-based explanation for Bountiful or its relationship to Nahom. Bountiful has been viewed as simply impossible even by highly educated moderns. So how did Joseph get that [Page 174]so precisely right, nearly due east from Nahom with numerous details now verified, on his own? Nehem on a map, an insignificant detail among hundreds, does nothing to give us a semi-plausible explanation for Joseph’s creation of Lehi’s Trail. It just doesn’t fit the facts.

Even if the Athenaeum did have rare European maps of Arabia that contained the name “Nehem” or “Nehhm,” albeit not listed in the 1839 catalog, and even if they were there in time for Joseph’s 1829 visit, there is still the problem of Joseph’s access to the fledgling library and whatever exotic maps it may have had. Important information comes from the Rochester Athenaeum Collection at RIT:

The first meeting of the Athenaeum was held on June 12, 1829, and Nathaniel Rochester was chosen as the first president. For a $5 annual fee, individuals could use the Athenaeum’s space in the Reynolds Arcade building for private events. More importantly, however, they could use the organization’s collection of books and journals. These materials were not limited to the field of science, but spanned a variety of subject areas. On February 12, 1830, the Athenaeum was granted a charter from the State of New York, with the stated purpose of “cultivating and promoting literature, science and the arts.” [emphasis added]39

Whatever treasures the Athenaeum had or would one day have, they probably were not available to Joseph. Like a variety of other libraries in the US at this time, this was not a free public library where any farm boy could wander in and handle rare maps of Arabia, if one imagines that the Athenaeum had such things. Joseph had just recently struggled to get money to buy paper for the translation process, and had to interrupt the translation in order to vainly seek for work in the area. He and Oliver had also been short on food. Joseph and Oliver were rescued from their dire impasse by Joseph Knight, Sr., who brought them food and enough paper to finish the translation.40 Joseph was relying on a mortgaged farm from Martin Harris to pay the overwhelming costs of printing the Book of Mormon. It would seem improbable that he would be ready to spend $5 in 1829 dollars to pay an annual fee to access a fledgling library that he had nearly no time to enjoy. The possibility that a member could have invited him in for free can also be discounted, as it again seems improbable that established citizens of Rochester would put their reputation on the line by inviting the widely derided farm boy as their personal guest into the elite library. (Indeed, there is no evidence that Joseph frequented libraries of any kind, and, as Robert Paul noted, “it [Page 175]is likely that during the 1820s he simply was not a part of the literary culture, that portion of the population for which books provide a substantial part of its intellectual experiences.”41) The Athenaeum is simply not a promising candidate for Book of Mormon origins. As with other maps RT has proposed to explain Lehi’s Trail, the proposed maps appear to be both inadequate and relatively inaccessible.

Of course, the contents of the Athenaeum are of no interest for Grunder’s basic theory. Could the Reynolds Arcade itself have played a pivotal role, along with the nearby aqueduct and its iron railing?

In the Nick of Time? June or July for Joseph’s Rochester Visit

The “nick of time” part for Grunder’s theory is one of several problem spots. Did Joseph actually visit Rochester before he had completed Lehi’s dream in the early chapters of 1 Nephi? June was a busy month for Joseph and I don’t think there is adequate time in Grunder’s scenario for a June Rochester trip followed by frenetic translation of almost the entire small plates of Nephi. First note that chronologies of the translation of the Book of Mormon put completion around July 1. For example, David Whitmer said that: “The translation at my father’s farm, Fayette Township, Seneca County, New York occupied about one month, that is from June 1, to July 1, 1829.”42 According to John Welch, based on an extensive compilation of evidence, nearly all of the 1830 Book of Mormon was produced at a remarkable pace in early 1829, beginning in April with the speech of King Benjamin in the book of Mosiah, finishing the book of Moroni by May, and then translating the small plates (1 Nephi-Omni) and the Words of Mormon before the end of June. “Virtually no excess time existed during those three months for Joseph Smith to plan, to ponder about, to research, to hunt for sources, to organize data, to draft, to revise, or to polish the pages of the original manuscript of the book.”43

On June 11, Joseph, possibly through the agency of Martin Harris, applied for a copyright for his book to help protect his rights, a process that required filing the printed title page of the Book of Mormon in a distant copyright office in Utica, New York, about 120 miles from Palmyra, as detailed by Michael Hubbard MacKay and Gerrit J. Dirkmaat.44 The title page of the 1830 Book of Mormon makes it fairly clear that the account of Ether and the burying or sealing up of the plates had already been described,45 so it seems fair to say that the translation of 1 Nephi was already underway by that date. A key question is when did Joseph go to Rochester and how much remained to be translated when he went?

[Page 176]Grunder depends on Joseph taking his time to get 1 Nephi started. He requires Joseph to have pretty much stopped translating after hitting the end of the Book of Mormon and its title page (apparently at the end) in order to seek out printers, before rushing to complete the last few pages. How many pages? There are 143 pages from 1 Nephi 1 to Omni in the 1981 printing of the Book of Mormon. Translation rates have been estimated at eight pages a day.46 During June, Joseph would also deal with the three witnesses, he would travel to Palmyra and then Rochester and spend time seeking printers, he would travel back to work with scribes to translate the plates, and then he would need at least half of June to complete the translation at a rapid pace. It’s no wonder that Grunder states that Joseph must have gone to Rochester early in June and then did the translation of 1 Nephi afterwards:

The latest comparison of original sources suggests that Joseph Smith and Oliver Cowdery were not settled in the Whitmer cabin to begin this part of the dictation until about June 5 (EMD 5:417, detailed chronology assembled from extensive documentation). Very shortly thereafter, they visited the Grandin printing shop in Palmyra. Then Joseph went on to Rochester where he was reported again almost immediately with Martin Harris.47

The negotiation with printers did not initially require Joseph to abandon the work of translation, for he sent Martin Harris to Palmyra “by early June, and possibly before” with a manuscript copy of the title page to use in negotiations.48 Martin met with Egbert B. Grandin in Palmyra. The man who became the typesetter, John Gilbert, reported that it was in early June when Harris and Grandin met.49 Grandin was skeptical and refused to take on the project. Grandin would publish an article on June 26, 1829 mocking the Book of Mormon project as the “result of gross imposition, and a grosser superstition,” showing that at this time in late June, Grandin was not seriously considering taking on the publication task.50 After Grandin’s rejection, Joseph and Martin together sought help from others in Palmyra, without success.

According to an employee of E.B. Grandin, Pomeroy Tucker, when the initial negotiations took place in June, Joseph brought the title page and some manuscript pages and was able to tell Grandin how many folios (sets of folded pages) would be needed to complete the book:

In June, 1829, Smith and the prophet [sic], his brother Hyrum, Cowdery the scribe, and Harris the believer, applied to [Page 177]Mr. Egbert B. Grandin, then publisher of the Wayne Sentinel at Palmyra (now deceased), for his price to do the work of one edition of three thousand copies. Harris offered to pay or secure payment if a bargain should be made. Only a few sheets of the manuscript, as a specimen, with the title-page, were exhibited at this time, though the whole number of folios was stated, whereby could be made a calculation of the cost. Mr. Grandin at once expressed his disinclination to entertain the proposal to print at any price, believing the whole affair to be a wicked imposture and a scheme to defraud Mr. Harris, who was his friend, and whom he advised accordingly.51

This suggests that the manuscript, of course, was nearly complete and Joseph at least knew how many more pages of text would be needed to complete the translation. Is this consistent with theories that suggest Joseph was ready to start creating major, lengthy new sections on the fly? Yet it appears there may still have been some translation to be done, so some additional content may have been forthcoming in the final days of June.

An important question at this point is when did Joseph go to Rochester to look for other printers to take on the task of publication? Pomeroy Tucker states that Joseph and his team “immediately” went to Rochester after visiting Grandin,52 but Tucker probably would not know the details of their trip apart from what Joseph would later tell Grandin sometime after his return. Of course, given early June negotiations with Grandin, one can assume that the trip to Rochester happened shortly thereafter, giving a mid-June estimate for that trip, which is what some authors have accepted.53

More recently, however, MacKay and Dirkmaat in From Darkness Unto Light state that Joseph Smith and Martin Harris decided to visit printers in Rochester, “likely arriving in Rochester sometime in July.”54 After several days discussing and negotiating with printers in Rochester, Elihu Marshall agreed to take on the project. This was not yet a good solution for Joseph, though, who would have a hard time staying close to the work in a town almost 25 miles from Palmyra, but the offer from Marshall gave him standing to renegotiate with Grandin, who now realized that someone was going to print to the book after all, and he might as well be the one to get the work, but under rather harsh terms that he would impose. According to MacKay and Dirkmaat, “While it is not known definitively when the men settled on terms with Grandin, by 11 August 1829, Jonathan Hadley reported in his paper that the [Page 178]Book of Mormon was ‘soon to be put to press’ in Palmyra rather than in Rochester.”55 The deal with Grandin was likely settled in late July or early August.56

In John Welch’s detailed review of the production of the Book of Mormon, the visit to Rochester is assigned to July 1829.57 A chronology at FairMormon also puts the Rochester visit in July 1829,58 with the Grandin deal being finalized in August. Further, in the widely cited and detailed Book of Mormon chronology compiled by Eldon Watson,59 the Rochester trip does not appear to take place in June at all, which is packed with Book of Mormon translation work. In that chronology, 1 Nephi 11 is completed by June 7, 1829. Later, 2 Nephi 27, giving details about the three witnesses, is estimated to be translated on June 20. It was the translation of 2 Nephi 27 with its mention of witnesses to the Book of Mormon that gave rise to the three witnesses event near the end of June. Whether Rochester was visited in mid-June or in July, Watson’s chronology leaves no room for speculating that something on that trip was a catalyst for material in 1 Nephi 8 and 1 Nephi 11. Lehi’s vision was already in ink.

If a bid from Elihu Marshall had been obtained in mid-June, why the lengthy delay in getting back to renegotiate with Grandin? Thanks to Marshall, Joseph had won an all-important competitive bid that he could leverage to make a deal with a printer much closer to home where the security of the manuscripts and the details of the work could be adequately supervised. If the issue of finalizing the printing plans was important enough for Joseph to delay the translation project in mid-June, why not follow-up immediately with Grandin upon returning from Rochester?

Arriving in Rochester in July means that Joseph wasn’t interrupting his urgent translation work to travel to Rochester. It would mean that he was probably done with the translation and would be able to soon provide the initial pages of the manuscript (which Oliver would be working on rapidly in July, producing the Printer’s Manuscript) once the printer was secured. In this scenario, if accurate, no matter how impressed Joseph was by the 4.5 stories of the Arcade, or any other tall building in Rochester, complete with nearby iron rod, a river, and fruit trees in the region, it would be too late to start dreaming about how to use that material in Lehi’s vision.

Of course, if Joseph did not go to Rochester in June, one could speculate that he made an earlier unrecorded trip to Rochester, or that an ally went there and created the story for him. Neither possibility [Page 179]provides a satisfying or evidence-based approach, and neither overcomes the many other deficiencies in Grunder’s theory.

An Early June Visit Fails to Solve the Timing Problem: The Significance of the Lost 116 Pages and Book of Mormon Intertextuality

Even if Joseph did visit Rochester in early June of 1829, early enough to have preceded the dictation of Lehi’s vision in 1 Nephi 8, the “nick of time” problem still isn’t resolved for Grunder’s theory or for RT’s twist on it (the proposal that Joseph found a rare map of Arabia in the Arcade). Making up the content of the books of Nephi at this late stage to incorporate newly encountered scenes from Rochester leaves us with significant problems. First, the record of Lehi, which was in the 116 lost pages that could turn up at any time, as far as Joseph knew, most likely contained some aspects of Lehi’s vision, for it is in the midst of Lehi’s discussions after his dream and just before Nephi’s own version of that dream that Nephi tells us that the many details of Lehi’s preaching at this time are given in the large plates (1 Nephi 10:2, 15). Nephi also tells us in the midst of Lehi’s dream-related account in 1 Nephi 8:29 that he is not going to write all the words of his father on this matter, which follows 1 Nephi 1:17 where Nephi explains that he is abridging the record of his father and then will give his own record. The lost 116 pages, apparently largely from Lehi’s record, can be expected to provide more details from Lehi’s visions and preaching than we find in Nephi’s abridgement, so we can expect the account of the rod of iron and the great and spacious building to have been included in the lost pages in some form, and thus to have already been committed to ink early in the Book of Mormon translation process. The same should apply to details of life and struggles along Lehi’s trail, including details that one might allege could come from a map.60

This is a point to emphasize. The material about Lehi’s vision and Lehi’s journey was very likely already on the lost 116 pages that Martin Harris had taken and shown to others. If Joseph were a con man making things up and fooling his scribes, Lehi’s vision — and the gist of the travels through Arabia — cannot be freshly concocted at this stage or else his primary source of funding and whoever may have seen the 116 lost pages could cry foul. Innovations from a mystery map in the Arcade don’t help, nor does inspiration from four floors of great spaciousness at the Reynolds Arcade. None of this is in the nick of time in any scenario.

[Page 180]Of course, postponing the proposed discovery of a map of Arabia to the very end of the Book of Mormon translation project resolves none of the many problems that have been noted with RT’s basic “Joseph had a map” theory and may only exacerbate them. Further, if Lehi’s vision, the details of Lehi’s trail, and other elements presented in 1 and 2 Nephi were devised by Joseph near the end of the Book of Mormon “translation” project, how does one account for allusions to such material elsewhere in the previously translated text? The intertextuality of Book of Mormon records linking its later writings to the writings of Nephi greatly reduces the plausibility of Grunder’s theory.

Many details in Nephi’s writings are relied on in subtle ways throughout the Book of Mormon, such as Lehi’s and Nephi’s use of dust imagery, building on the theme of rising from the dust in Isaiah 52, which is fittingly used by Moroni to close the Book of Mormon and is employed in other subtle ways in the text. Walter Brueggemann has shown that the Old Testament uses the concept of rising or returning to the dust to deal with themes of kingship and enthronement as well as resurrection and keeping divine covenants.61 In light of Brueggemann’s work, David Bokovoy explores Lehi’s call for his sons to “rise from the dust” and sees Nephi’s response as being carefully constructed to buttress his claim to spiritual and political authority.62 Recognizing that Nephi and Lehi gave great emphasis to Isaiah 52, where we find Isaiah’s call to “rise from the dust,” we can better understand the significance of Moroni’s closing words, which again cite Isaiah 52, and see the influence of Isaiah 52 as a major source of Nephite teaching woven into other sections of the Book of Mormon. In a recent essay, I argue that understanding the dust-related themes introduced by Lehi and Nephi, and affirmed by Jacob, provides even more unity to the Book of Mormon than Bokovoy identified. For example, it helps make better sense of the puzzling question posed by King Noah’s priests to Abinadi in an effort to challenge his teachings. It also can strengthen our appreciation of the artistry of Alma 36 as an even richer work of Semitic poetry than previously appreciated.63

While the iron rod is not explicitly mentioned later in the Book of Mormon, several concepts related to Lehi’s vision are present elsewhere, including:

Many aspects of 1 Nephi and 2 Nephi and the experiences and teachings along Lehi’s trail are artfully woven into the Book of Mormon (just as the vision of Lehi and Nephi is later woven into Nephi’s own concluding words at the end of 2 Nephi65). The brass plates retrieved from Laban play a critical role. The Liahona plays an important role (see Alma 37). The basic story line with Nephi, Lehi, Laman, Zoram, etc., is integrated throughout the book in numerous references, as is the basic idea of their exodus from Jerusalem in a difficult trek that would take them to the New World where the Nephites will again apply the name Bountiful from Nephi’s account.

One intriguing example of the pervasive intertextuality of Nephi’s writing with the rest of the Book of Mormon was recently reported in Matthew Bowen’s “Alma — Young Man, Hidden Prophet” at The Interpreter.66 Bowen explains several apparent Hebraic wordplays involving the name Alma (once mocked as an obviously female name [Page 182]from Latin or Spanish, but now known to be an authentic ancient Hebraic name for a man, attested by archaeological work long after Joseph Smith passed away67), and then notes how the wordplays appear to deliberately parallel Nephi’s apparent wordplay on his own name, which can mean “good” or “goodly” in Egyptian. He writes:

Indeed, the aforementioned wordplay on “Alma” as a “young man” becomes even more striking in view of earlier wordplay in the Book of Mormon. For example, when we compare the biographical introduction of Alma to his ancestor Nephi’s autobiographical introduction and a related biographical description of himself and why his faith diverged from his brothers, the textual dependence of Alma’s biography on Nephi’s autobiography is clear:

 

1 Nephi 1:1; 1 Nephi 2:16

Mosiah 17:2

I, Nephi,
having been born of goodly parents , therefore I was taught somewhat
in all the learning of my father [ʾăbî]
and having seen many afflictions
in the course of my days,
nevertheless, having been highly favored
of the Lord in all my days;
yea, having had a great knowledge
of the goodness and the mysteries of God, therefore I make a record of my proceedings in my days. (1 Nephi 1:1)

 

 

 

But there was one among them whose name was Alma ,
he also being a descendant of Nephi .
And he was a young man ,
and he believed the words
which Abinadi had spoken ,
for he knew concerning the iniquity
which Abinadi had testified
against them;
therefore he began to plead with the king that he would not be angry with Abinadi, but suffer that he might depart in peace.

And it came to pass that I, Nephi ,
being exceedingly young ,
nevertheless being large in stature,
and also having great desires to know
of the mysteries of God ,
wherefore, I did cry unto the Lord;
and behold he did visit me,
and did soften my heart
that I did believe all the words
which had been spoken by my father [ʾăbî]; wherefore, I did not rebel against him
like unto my brothers. (1 Nephi 2:16)

On one hand, the onomastic wordplay on Alma and “young man” imitates the autobiographical wordplay of 1 Nephi 1:1 (on Nephi and nfr, “good[ly],” “goodness,” etc.). On the other hand, the wordplay cunningly incorporates Nephi’s subsequent autobiographical description of himself as “young.” Beyond this, the two texts from Nephi’s personal

[Page 183]writings share numerous lexical connections that Mosiah 17:2 utilizes — e.g., Nephi, knowledge/know, my father, “taught in all the learning of my father”/“did believe all the words which had been spoken by my father”….68

This level of intertextuality, with Semitic wordplays on one man’s name being patterned on the introduction from Nephi, is difficult to explain using theories based on fabrication by Joseph Smith, with Nephi’s record being spewed out on the fly at the end of the fabrication process. It would be difficult to explain even if Joseph were a well-educated scholar with abundant time and resources at his disposal, with a text that was worked out slowly over years and not simply dictated from a hat.

Loren Blake Spendlove and Tina Spendlove also recently discussed a relationship between the words of Christ to Alma and the words of Lehi in discussing the tree of life:

Lehi’s dream, in the initial chapters of the Book of Mormon, focuses on Lehi’s desire that his family members come to the Tree of Life — to that same tree that was in the center of the Garden of Eden, our place of origin. It is interesting to observe Lehi’s persistent desire that his family “come unto me and partake of the fruit” (1 Nephi 8:15, 16, and 18). Lehi’s words echo those of Christ himself as he spoke through the prophet Alma: “Repent and I will receive you. Yea, he saith: Come unto me and ye shall partake of the fruit of the tree of life; yea, ye shall eat and drink of the bread and the waters of life freely. Yea, come unto me and bring forth works of righteousness, and ye shall not be cut down and cast into the fire” (Alma 5:33–35).69

Alma 5 appears to draw upon several aspects of Lehi’s vision, including his specific words, “come unto me and partake of the fruit,” the water of life, and the concept of judgment and the opposing elements of salvation and damnation that are involved in his vision, which strengthens the intertextuality in the Book of Mormon related to Lehi’s dream. While these elements are also biblical concepts, they are thoroughly at home in a variety of contexts in the ancient Near East and its large body of tree of life literature, as discussed below.[Page 184]

The More Serious Timing Problem:
Improvising an Ancient Text

Grunder’s theory makes 1 and 2 Nephi a hastily composed text spewed out on the fly by Joseph in the last days of his Book of Mormon project, inspired in part by his encounter with scattered elements in Rochester. For those who have examined the literary strengths of Nephi’s writings, Grunder’s theory may well seem absurd.

As early as 1972, the structural sophistication of Nephi’s writings, including overarching chiasmus, was already noted.70 Modern Bible scholars exploring the origins of biblical texts routinely point to the significance of doublets — instances where a story element appears to be repeated unnecessarily — as important clues about origins of the text. For some scholars, doublets are routinely considered to indicate that two different sources have been patched together by a redactor, and this line of thought has been important in development of the so-called Documentary Hypothesis.71 For example, the pair of creation stories in Genesis 1 and 2, the two versions of the flood story in Genesis 6–9, and two apparent versions of David’s introduction to Saul’s court in 1 Samuel 13 and 15 have been considered as evidence of redaction from different incompatible texts. On the other hand, some scholars consider the possibility of higher literary purposes in such doublets, and have argued that the double occurrence of some elements is due to the influence of ancient narrative and poetical tools in which repetition can play an important role that is foreign to modern sensibilities. Robert B. Chisholm has provided detailed arguments that the “parade examples” of doublets used to support the Documentary Hypothesis, namely the accounts of the flood and David’s introduction to Saul’s court, actually show literary unity with the doubled elements and appear to be the result of deliberate choice by the author.72 G.J. Wenham has proposed that chiasmus in the flood account provides strong evidence for the unity of the account and its doublets, whether it is present as originally drafted or has been redacted to provide the unity that Wenham sees.73

The use of chiasmus in particular has been used by several scholars to suggest that there may be literary unity in some passages long assumed to be fragments from two or more disparate sources. The presence of the obvious doublet in Lehi’s vision, with Lehi’s version in 1 Nephi 8 being presented followed by Nephi’s vision in 1 Nephi 11 and a discussion of the meaning of the tree of life in 1 Nephi 15, may strike a casual modern reader as redundant, scattered, and rambling, as one might expect from a novice dictating a poorly considered story on the fly, in a hurry, with [Page 185]little opportunity for revision. However, there may be significant design and structure present, as John Welch showed in an early article on chiasmus on the Book of Mormon:74

 

Welch offers this commentary on the significance of the chiastic structure, and responds to those who will say it is merely contrived and accidental:

More than ever before, we are now in a position to admire the purposeful and stunning unity of Nephi’s message. The long-neglected principle of chiasmus unfolds the fact that when Nephi revised the record of Lehi, Nephi’s mind was clearly organized and his heart was intent upon expressing the central importance of his great vision with the Spirit of the Lord. He achieves this expression via chiasmus. …

Should we consider it contrived that Ishmael is mentioned only twice in the entire Book of Mormon and that these two occurrences just happen to fall symmetrically around 1 Nephi 11? How else, except by chiasmus, can we explain the postponed interpretation of the vision of the tree of life? One would expect the interpretation to follow immediately after the dream, as most interpretative passages in the Book of Mormon do, and not several chapters later.

[Page 186]Are we to believe that the unruly brothers of Nephi really waited nine chapters to marry the daughters of Ishmael? Are we to neglect such specific parallels between the first half of 1 Nephi and its second half — e.g. 3:7 and 17:3 — or again the fact that Nephi is bound by cords once in chapter 5 and again in chapter 18? Or how are we to explain the fact that Nephi wrote two books (1 Nephi and 2 Nephi) instead of just running it all together into one, except by reference to the individual structure of each book?

To answer any of these questions, chiasmus must be called into the discussion to explain the underlying structural organization behind Nephi’s written record.

Fortunately, it is a perfectly natural thing to appeal to chiasmus in cases such as these. After all, if Lehi were the one who was telling the story, he would have told it much differently. Would Lehi have spent so much time on the story of Laban and so little time on the vision that prompted him to get out of Jerusalem? Surely not. What we have in 1 Nephi is Nephi telling the story, and he does so in Nephi’s way of seeing things. Thus it is perfectly natural to find that Nephi gives his autobiography a structure all its own, a structure that conveys by its very form a message of emphatic centrality and symmetrical contrasts about important events in Nephi’s life.75

Noel Reynolds has also observed that there is significant literary structure in Nephi’s writing that helps illustrate his purposes and his superb craftsmanship in creating 1 Nephi.76 Reynolds points to textual clues showing that Nephi has deliberately split 1 Nephi into two parts. After noting that Nephi’s selection of episodes in his account appear to be designed to support his overarching theme stated in 1 Nephi 1:20, that he will show us the “tender mercies of the Lord,” Reynolds writes:

Yet further analysis reveals a far more complex structure. At the beginning of the book, Nephi explains that he will first make an abridgment of his father’s record, then an account of his own doings. Beginning at chapter 10, he states that he will now commence with an account of his own proceedings, reign, and ministry. At the end of chapter 9, as at the end of chapter 22 (the last chapter in 1 Nephi), Nephi concludes with a restatement of his thesis, punctuated by the formal ending, [Page 187]“And thus it is. Amen.” The suggestion seems to be that there are two records, an abridgment of Lehi’s record followed by an account of Nephi’s proceedings, but if those few verses were removed, we would never suspect two records. The story is continuous; Nephi is the narrator of the entire book from beginning to end. And the very next verse continues the speech of Lehi that was interrupted to end chapter 9. We know of Lehi’s teachings through Nephi’s report, not through a condensation of Lehi’s own record. So why does Nephi divide the book in this seemingly arbitrary manner?77

Based on his detailed analysis, Reynolds shows that Nephi’s record in 1 Nephi is not two distinct texts that have been joined together but rather is one unified text drawing upon several sources (Nephi’s recollection and writings, Lehi’s writings, and the brass plates) that has been crafted to have two parallel parts that achieve a masterful literary structure. The first part, 1 Nephi 1–9, which Reynolds labels as “Lehi’s account,” has detailed pairings with the episodes in the second part, 1 Nephi 10–22, “Nephi’s account,” showing evidence of deliberate and even intricate pairing (see especially Reynolds’ Table 1 and Table 278). For example, the stories told for the seventh item in each of these two sections, the trip to bring back Ishmael and his family in the first half and the journey to the ship in the second half, respectively, can each be broken into eight elements that are presented in exactly the same order in both parallel accounts.79

In addition to this broad parallelism, Reynolds finds significant chiastic structure (introverted parallelism) in 1 Nephi, with both the first and second parts showing chiasmus that focuses on key historical elements in both halves, the retrieval of the brass plates and the construction of the ship, respectively. He also finds other aspects of the structure that appear to have been tailored to create chiasmus. Reynolds finds intricate structure, including several forms of parallelism, in 1 Nephi, while 2 Nephi appears more to be a collection of additional theological discourse and other items without the same level of structure.80 More recently, Dennis Newton has shown that 15 relatively compact, multi-level chiasms in Nephi’s writings feature high levels of thematic unity and craftsmanship with significant evidence of intent.81 The sophisticated literary structure in 1 Nephi, rich in ancient Hebraic forms of parallelism that were not widely appreciated in Joseph’s day, is inconsistent with a few days of hasty dictating from a hat by young [Page 188]Joseph Smith, awestruck by the wonders of Rochester and trying to shoehorn some bold new images into his book at the last minute.

Joseph’s Hat Trick? What Scholarship Reveals about Nephi’s Writings

The theory that Joseph fabricated Nephi’s writings based on Joseph’s environment, dictated in a hurry from a hat, becomes all the more implausible when one considers the treasures that scholars have uncovered in Nephi’s artfully composed works. Time after time his works reflect the skill of an ancient craftsman. Based on a partial sampling of such finds, to fabricate Nephi’s works you would need to

  1. Create Nephi’s psalm in ways consistent with ancient patterns of psalm writing that would be studied and expounded long after the fabrication. For example, using form-critical analysis, Matthew Nickerson has shown that Nephi’s psalm fits the pattern of the “individual lament” with five elements: 1) invocation, 2) complaint, 3) confession of trust, 4) petition, and 5) vow of praise.82 It has also been the subject of scholarly analysis of its eloquent poetical elements that are consistent with ancient Semitic authorship83 and for its adherence to covenant patterns in the Psalms.84 Of course, critics can argue that Nephi’s lyrical psalm could be fabricated in modern times by one thoroughly steeped in the Bible.
  2. Use chiasmus not only for the overall structure of 1 Nephi but also for multiple sections of your writings. For example, Nephi’s psalm has the chiasmus a) my soul delighteth, b) my heart pondereth, c) great goodness of the Lord, c’) great and marvelous work, b’) my heart exclaimeth / my heart sorroweth, a’) my soul grieveth (2 Nephi 4:16–17). Of course, critics can argue that chiasmus is accidental or the result of osmosis from the Bible without any need for Joseph to have understood this ancient poetical form.
  3. Imbue the tree of life description with significant allusions to ancient Wisdom themes from pre-exilic Israel,85 including association of the tree with a woman (in Nephi’s case, Mary, the mother of Christ).86
  4. Use “up” and “down” in referencing travels around Jerusalem that accurately and consistently reflect the topography.87
  5. [Page 189]In quoting Isaiah, make subtle changes that reflect a sophisticated knowledge of Hebrew, years before you begin studying Hebrew.88
  6. Build multiple ancient Hebraic and Egyptian wordplays into your text, frequently with great relevance to the message. Such wordplays include the name of Nephi itself and the way it is used in the Book of Mormon,89 the association of Nahom with the mourning over the death of Ishmael,90 wordplay involving the name Joseph,91 wordplay on the name Mary in 1 Nephi,92 and several instances of Hebraic wordplay in the Book of Enos.93 There is also the possible Egyptian wordplay involving the word of God as a rod, discussed below.
  7. Integrate exodus themes and other ancient Jewish themes into your account in a way that shows rich and often subtle artistry firmly anchored in antiquity.94 One example is the way that Nephi’s description of the slaying of Laban is patterned after the account of David and Goliath, thereby serving as an important basis for Nephi’s descendants in recognizing the validity of Nephi’s claim to be the rightful ruler of the people, as Ben McGuire’s work demonstrates.95
  8. Create the sense of decades of maturing in a real character, Nephi, whose perspectives and objectives change over the course of his writings in a way that is consistent with what can be inferred from the text regarding the timeline of his writings.96
  9. Write Nephi’s section with a voice distinctly different than Mormon and Moroni’s, the voices that had just been used in the previously written Book of Mormon sections.97 In fact, make that voice reflect what Nephi claims to be: a Hebrew man steeped in Jewish scripture, thought, and symbolism, from the viewpoint of a conservative religious devotee opposing religious apostasy in the Jerusalem of 600 bc, accurately reflecting the religious tensions between traditional worshippers and Deuteronomist reformers in pre-exilic Israel.98 Do it so thoroughly that it will impress a modern non-LDS scholar specializing in that topic.99
  10. Introduce numerous details in the story that will seem like anachronisms but will later be validated as plausible elements. Examples include the idea of writing Hebrew in some form of Egyptian,100 the concept of ancient scripture [Page 190]on metal plates,101 Laban’s steel sword and its gold hilt,102 Lehi’s practice of offering sacrifices,103 storing scriptures in a treasury,104 and so forth.
  11. And don’t forget to describe the journey across Arabia in ways that would seem ridiculous in 1830 and only become plausible over a century later, with remarkable finds relating to geographical details for places like the River Laman and Bountiful, with accurate and plausible directions, and even future archaeological finds verifying the ancient authenticity of an important name of the area where Ishmael was buried, a name obviously obtained from locals.105

This list doesn’t even include the most fundamental aspects of the case for the authentic nature the Book of Mormon, such as the diverse witnesses of the gold plates and their reliability, the witnesses of the translation process, and other extensive internal and external evidences.106 Grunder’s theory or any extensions thereof do not begin to address these issues. They are thus completely lacking explanatory power.

Weighing the Iron Rod: Modern Architectural Element or Ancient Symbol?

Grunder’s focus on modern elements, like conventional iron railings and modern aqueducts, fails to consider the strengths of the Book of Mormon that emerge when we give it a chance and consider its connections to the ancient world. RT feels the iron rod is anachronistic, and is grateful for Grunder’s work that clarifies the source of that Book of Mormon element. But both men are overlooking rather noteworthy ancient connections that might better clarify the Book of Mormon on this point.

Before assuming that the Book of Mormon is referring to a modern iron railing, we should consider the iron rod in the context of the Book of Mormon as well as the Bible. Joseph was presumably familiar with railings and fences. Why not describe the rod as such? “Rod” is not a common way in modern English to describe the function of what we perceive as a common railing in Lehi’s dream. But it is an appropriate word for an ancient Semitic text.

Anciently, a rod conveyed the meaning of authority and divine power. In the Old Testament, the word “rod” is introduced in Moses’ encounter with God on Sinai, when the Lord asks a significant question: “And the LORD said unto him, What is that in thine hand? And he said, [Page 191]A rod” (Exodus 4:2). The rod of Moses would become a tool for smiting enemies or overcoming the barrier to liberty and bringing the Israelites to the promised land, just as the rod in Lehi’s dream brings us to the tree of life. Interestingly, Christine Meilicke observes that Moses’ staff is almost always associated with some kind of water (Nile, sea, well in the wilderness), just as the rod of iron protects travelers from a filthy river (waters of death) and leads them to the waters of life from the tree of life.107 She also observes that the staff of Moses and the blooming rod of Aaron, another symbol of authority, are often blended into one.108

The rod can be used as a weapon to thwart or subdue enemies of God, as in Psalm 2:9 and the smiting rod in Isaiah 10:24 and 11:4. In the latter verse, the Lord “shall smite the earth with the rod of his mouth,” showing a connection to the role of the rod as the “word of God.” Similar action against the wiles of the adversary is also a function of “the word of God” (in context, arguably the rod) in Helaman 3:29. The rod from the stem of Jesse is a Messianic symbol (Isaiah 11:1). All these uses provide relevant context for the significance of the rod, as it might have been understood in Nephi’s world.109

In the Book of Mormon, Nephi first uses “rod” in an interesting scene in 1 Nephi 3:28–29, where the rod is used both as a tool for smiting and implicitly as a symbol of authority:

And it came to pass that Laman was angry with me, and also with my father; and also was Lemuel, for he hearkened unto the words of Laman. Wherefore Laman and Lemuel did speak many hard words unto us, their younger brothers, and they did smite us even with a rod.

And it came to pass as they smote us with a rod, behold, an angel of the Lord came and stood before them, and he spake unto them, saying: Why do ye smite your younger brother with a rod? Know ye not that the Lord hath chosen him to be a ruler over you, and this because of your iniquities? Behold ye shall go up to Jerusalem again, and the Lord will deliver Laban into your hands. [emphasis added]

The angel not only spares Nephi’s life, but specifically questions the use of a rod by the wicked brothers. The question isn’t merely “Why do ye smite your younger brother?” but why do they smite him with a rod? This is followed by a challenge to their leadership status: “Know ye not that the Lord hath chosen him to be a ruler over you and this because of your iniquities?” The right to wield the rod is Nephi’s, not his elder brothers’. [Page 192]Here the rod is a misused symbol of authority as well as a smiting tool, consistent with ancient Near Eastern perspectives.110

In that context, I suggest it is improper to neglect what Nephi and other scriptures (e.g., Psalm 2:9; Isaiah 10:24, 11:4; cf. the much later Revelation 2:27) already have told us about the symbol of the rod when we encounter it again in Lehi’s dream. Obviously the rod, however it was portrayed, was much longer than a typical hand-held rod, staff, or scepter. It extended along a bank and led to the tree of life. But that doesn’t necessarily make it a modern railing held up by periodic vertical shafts.

In the Book of Mormon, those who gained the benefits of the rod “caught hold of the end of the rod of iron” and then pressed forward by “clinging to the rod” (1 Nephi 8:24), and finally reached the tree of life by “continually holding fast to the rod of iron” (1 Nephi 8:30). The interaction with the rod seems to be one of grabbing and not letting go. This could be advancing along the rod, one grip or handhold at a time, but the language leaves open the possibility that the rod might have been extended toward people on the bank to then pull them toward the tree of life if they would but grab the end and hold on, contrary to the image we tend to have of moving along the rod as we do with a conventional railing. Perhaps the rod as “word of God” played a more dynamic role in leading, guiding, and shepherding people (see the quote from Margaret Barker below on this idea), while also being able to “divide asunder” the cunning, the snares, and the wiles of the devil as does the word of God in Helaman 3:29, to “land their souls” in the kingdom of heaven (Helaman 3:30). In any case, it’s a dream and elements don’t have to have normal dimensions and properties.

Nephi continues using the word “rod” in his writings. In 1 Nephi 17:41, he refers to an active use of the rod to “straiten” the Israelites in the wilderness as he juxtaposes the rod of Moses with the story of the brass serpent on a pole:

And he did straiten them in the wilderness with his rod; for they hardened their hearts, even as ye have; and the Lord straitened them because of their iniquity. He sent fiery flying serpents among them; and after they were bitten he prepared a way that they might be healed; and the labor which they had to perform was to look; and because of the simpleness of the way, or the easiness of it, there were many who perished.

The rod of Moses, famous for its association with serpents in Exodus 4, is linked here with the brass serpent on a (rodlike?) pole, [Page 193]and the overall effect is to “straiten” the Israelites, or to guide them on a strait (narrow) course that, like the yoke of Christ, is easy but often rejected. Here the rod, the Messiah, and the straight and narrow path are associated. Later uses of “rod” by Nephi are in quoting from the Old Testament, where the smiting action of the rod is mentioned several times (2 Nephi 20: 5, 24, 26; 21:4, 24:29, 30:9).

In 2 Nephi 3:17, the rod as a symbol of power is found in a prophecy of the Lord given anciently to Joseph the son of Jacob and recorded on the brass plates, possibly in the Egyptian script or language that Joseph may have used: “I will raise up a Moses; and I will give power unto him in a rod; and I will give judgment unto him in writing.” In this couplet, the rod and writing are linked, possibly drawing upon the Egyptian language wordplay in which “rod” (mdw) means “words,” in line with the apparent wordplay in Lehi’s dream where the iron rod is explicitly identified as “the word of God.” On this matter, one of Matthew Bowen’s many notable contributions in Book of Mormon studies is recognizing the ancient Semitic wordplay apparently involved in Nephi’s identification of the iron rod as the word of God:

Further support for the antiquity of Nephi’s imagery is detectable in his own comparison of the word to a rod, a comparison that may involve wordplay with the Egyptian term for “word” and “rod.” Although we have the Book of Mormon text only in translation and do not know the original wording of the text, we can use our knowledge of the languages that the Nephite writers said they used — Hebrew and Egyptian (1 Nephi 1:2; Mormon 9:32–33) — to propose reasonable reconstructions.

We note that the Egyptian word mdw means not only “a staff [or] rod” but also “to speak” a “word.” The derived word md.t, or mt.t, probably pronounced *mateh in Lehi’s day, was common in the Egyptian dialect of that time and would have sounded very much like a common Hebrew word for rod or staff, matteh. It is also very interesting that the expression mdwntr was a technical term for a divine revelation, literally the “the word of God [or] divine decree.” The phrase mdwntr also denoted “sacred writings,” what we would call scriptures, as well as the “written characters [or] script” in which these sacred writings were written.

[Page 194]Now consider Nephi’s comparison of the word and the rod in the context of the Egyptian word mdw:

I beheld that the rod [mdw/mt.t, Hebrew matteh] of iron, which my father had seen, was the word [mdw/mt.t] of God. (1 Nephi 11:25)

And they said unto me: What meaneth the rod [mdw/mt.t, Hebrew matteh] of iron which our father saw, that led to the tree? And I said unto them that it was the word [mdw/mt.t] of God; and whoso would hearken unto the word of God, and would hold fast unto it, they would never perish. (1 Nephi 15:23–24)

An indication of Nephi’s awareness of the play on words is his use of the expression “hold fast unto” the “word of God,” since one can physically hold fast to a rod but not to a word (compare Helaman 3:29). Nephi’s comparison of the rod of iron to the word of God also makes very good sense in light of other scriptural passages that employ the image of the iron rod. But the comparison takes on even richer connotations when viewed as a play on multiple senses of the Egyptian word mdw. Since Lehi’s language consisted of the “learning of the Jews and the language of the Egyptians” (1 Nephi 1:2), we would reasonably expect that Lehi and his sons (Nephi in particular) were aware of, and probably even used, the common word mdw/mt.t in at least some of those senses. It seems unlikely that the word’s phonetic similarity to Hebrew matteh would have escaped their attention. On the contrary, it would plausibly explain Nephi’s apparent substitution of “word” for “rod” in later remarks to his brothers in 1 Nephi 17:26, 29: “And ye know that by his word [mdw/mt.t] the waters of the Red Sea were divided …. And ye also know that Moses, by his word [mdw/mt.t] according to the power of God which was in him, smote the rock, and there came forth water.”

Nephi’s imagery itself, along with its possible Egyptian language wordplay, further attests the antiquity of the Book of Mormon. Certainly Joseph Smith in 1829 could not have known that mdw meant both “rod” and “word.” However, Nephi, in the early sixth century bc likely had a good understanding of such nuances, and he may have employed [Page 195]them as part of a powerful object lesson for his brothers. [footnotes omitted, emphasis original]111

In fact, the Egyptian hieroglyph for “word” is the symbol of the walking stick, a rod.112 Further, Bowen observes in a footnote that Nephi’s introduction of the rod of iron may involve a polyptoton, in which words derived from the same root are used in a single sentence. Related to the Egyptian word for rod and word, mdw, is the Hebrew word maṭṭeh (מטה) meaning staff, rod, or shaft, which derived from the root NTH meaning to “stretch out, spread out, extend, incline, bend.” Thus, 1 Nephi 8:19 could be an interesting polyptoton: “And I beheld a rod [maṭṭeh] of iron, and it extended [nth] along the bank of the river, and led to the tree by which I stood.” Bowen also notes that an Egyptian transliteration of the Hebrew maṭṭeh (“rod”) and Egyptian mdw/mt.t (“rod, word”) would have been graphically similar or even identical if written in demotic characters.113

I find the potential wordplay around related Hebrew and Egyptian words to be highly interesting, difficult to attribute solely to another lucky guess from Joseph, and not the kind of thing one would think up on the fly after being impressed by an aqueduct in Rochester, or even with leisurely study in 1829.

Inherent in the wordplay and in the meaning of the iron rod is the link between the abstract concept of the word and a physical rod. This is also part of the previously mentioned intertextuality between 1 Nephi and Helaman 3, particularly vv. 29–30:

Yea, we see that whosoever will may lay hold upon the word of God, which is quick and powerful, which shall divide asunder all the cunning and the snares and the wiles of the devil, and lead the man of Christ in a strait and narrow course across that everlasting gulf of misery which is prepared to engulf the wicked —

And land their souls, yea, their immortal souls, at the right hand of God in the kingdom of heaven, to sit down with Abraham, and Isaac, and with Jacob, and with all our holy fathers, to go no more out. [emphasis added]

Here language is used that echoes Nephi in several ways. In addition to laying “hold” on the word of God, something one can physically do with an iron rod but not to words themselves, we learn that the word, like the iron rod, serves to lead one in a straight course to eternal life (similar to the tree of life) and to avoid the “gulf of misery” that Nephi [Page 196]also speaks of (2 Nephi 1:13, possibly building on the “terrible gulf” of Lehi’s dream in 1 Nephi 12:18 and the “awful gulf” of 1 Nephi 15:28; cf. Alma 26:20 and Helaman 5:12). The dangerous journey to eternal life is made possible if one will “lay hold upon” the word of God and pursue its straight and narrow course. The iron rod theme seems to have been part of background in Helaman 3, and thus not readily explained by something Joseph saw after dictating Helaman.

Consistent with Nephi’s usage, John Tvedtnes observes that the Old Testament links the voice of God with the concept of a rod:

The use of a rod to represent words or speech is found in Proverbs 10:13 and 14:3. In other passages, it refers specifically to the word of God. In Isaiah 30:31, “the voice of the Lord” is contrasted with the rod of the Assyrians. In a few passages, the rod is compared to a covenant with God which, like a rod, can be broken (Ezekiel 20:37; Zechariah 11:10, 14). Micah wrote, “The Lord’s voice crieth unto the city, and the man of wisdom shall see thy name: hear ye the rod, and who hath appointed it” (Micah 6:9). Isaiah wrote of the Messiah, “But with righteousness shall he judge the poor, and reprove with equity for the meek of the earth: and he shall smite the earth with the rod of his mouth, and with the breath of his lips shall he slay the wicked” (Isaiah 11:4).114

These connections are useful after the fact in examining the appropriateness of the iron rod as a symbol for the word of God, but seem inadequate to provide a basis for fabrication of that concept, particularly in light of the clever wordplay involved.

RT’s objection, mentioned above, was not to the use of a rod per se in the dream, but an iron rod. He states that “it is clearly not an ancient motif,” and observes that there were “no rods of iron set next to rivers” in Nephi’s time, which is correct as far as I know. Thus, he finds Grunder’s suggestions convincing, or at least “difficult to ignore.”115

Is a rod of iron a nineteenth century concept? Is it impossible to have been used in a divinely inspired vision in 600 bc? First realize that iron itself is not the problem. The Iron Age was well underway in Lehi’s day.116 Even the “fine steel” of Laban’s sword is not anachronistic,117 as some critics have claimed (especially those in the first few decades after the Book of Mormon, before the history of iron became better known), though high-quality steel could be rare and precious. So the problem raised by RT appears to be not the iron itself, but an iron rod as described by Nephi and Lehi. In saying it was not an ancient motif, perhaps RT [Page 197]refers only to iron rods next to rivers. Of course, in the Old Testament and related ancient records we find that there were iron knives, iron swords, iron tools, iron cups, iron beds, iron yokes, etc., and even iron rods. Psalm 2:9 specifically mentions an iron rod in a setting related to divine authority and the ruling or conquering of nations. Here is the context from verses 7–10:

Yet have I set my king upon my holy hill of Zion.

I will declare the decree: the LORD hath said unto me, Thou art my Son; this day have I begotten thee.

Ask of me, and I shall give thee the heathen for thine inheritance, and the uttermost parts of the earth for thy possession.

Thou shalt break them with a rod of iron; thou shalt dash them in pieces like a potter’s vessel.

Be wise now therefore, O ye kings: be instructed, ye judges of the earth. [emphasis added]118

The iron rod here is not just an aberrant kjv translation. The NIV also has “iron rod” while the NASB has “rod of iron.”

The kjv also mentions iron bars in Psalm 107:16 and Isaiah 45:2. The Hebrew word for “bars” is bĕriyach (ַבְּרִיח), Strong’s H1280, which can mean a crossbeam or bar used to connect wooden boards of the tabernacle or can be a bolt or bar for shutting doors or gates.119 Here we have an iron beam-like or rod-like object that appears to be horizontal, again suggesting that iron horizontal objects serving some kind of structural or barrier function would not be inconceivable to Lehi, though here the function of the rod is not to guide. Interestingly, in Isaiah 45:2, the iron bars are mentioned after stating that the Lord would make that which was crooked (crooked paths, apparently) straight.

Jeremiah 1:18 also speaks of an iron pillar: “I have made thee this day a defenced city, and an iron pillar, and brasen walls against the whole land….” The brass walls coupled with iron pillars (vertical rodlike elements?) defend the city. The Hebrew word translated here as “pillar” can also be a platform or scaffold,120 so could this include a fencelike function? Probably not. However, structural iron elements should not be unrecognizable to Lehi, including iron structures used to protect people, even if rare and expensive at the time.

Like RT, I am not aware of any ancient rivers in the Middle East that had iron rods along them, but that does not mean it could not have [Page 198]been an intelligible concept in a dream nor does it require that we look to modern sources for the concept of an iron rod. Given the presence of iron in Lehi’s day, the significance of rods, and the reference to an iron rod in Psalm 2, as well as other structural iron features in the Old Testament, iron in the form of a rod within a mere dream should not be overly puzzling.

Adding further credibility to the argument for ancient roots of the iron rod as portrayed in the Book of Mormon, non-LDS scholar Margaret Barker writes:

Consider as well the mysterious rod of iron in this Book of Mormon vision (1 Nephi 8: 20; 11: 25). In the Bible, the rod of iron is mentioned four times as the rod of the Messiah. Each mention in the King James Version says the Messiah uses the rod to “break” the nations (Psalm 2:9) or to “rule” them (Revelation 2: 27; 12:5; 19:15). The ancient Greek translation (the Septuagint) is significantly different; it understood the Hebrew word in Psalm 2:9 to mean “shepherd” and it reads, “He will shepherd them with a rod of iron.” The two Hebrew verbs for “break” and “shepherd, pasture, tend, lead” look very similar and in some forms are identical. The Greek text of the Book of Revelation actually uses the word “shepherd,” poimanei, of the Messiah and his iron rod, so the English versions here are not accurate. The holy child who was taken up to heaven (Revelation 12:5) was to “shepherd the nations with a rod of iron.” The King James Version of Micah 7:14 translates this same word as “Feed thy people with thy rod,” where “guide” would be a better translation. Psalm 78:72 has, “He fed them … and guided them,” where the parallelism of Hebrew poetry would expect the two verbs to have a similar meaning: “He led them … he guided them.” Lehi’s vision has the iron rod guiding people to the great tree — the older and probably the original understanding of the word.121

Let’s not make the mistake of projecting modern views of iron railings into Lehi’s dream and then finding that the iron rod is too modern to be from an ancient text. Iron rods, pillars, and bars are attested in the Old Testament and could have been known and recognizable to Lehi and Nephi, with symbolism and even linguistic aspects relevant to Nephi’s usage in an ancient era. Lehi’s dream and the rod of iron fits the ancient setting of the Book of Mormon better than a modern railing from Rochester in Joseph’s day.[Page 199]

The Tree of Life and the Whiteness of Its Fruit

The tree of life as portrayed in Lehi’s dream and in related references later in the Book of Mormon (e.g., Alma 5:33–36, 62; Alma 12:21–26; Alma 32:38–42; Alma 42:1–17) certainly draws upon biblical themes,122 but also resonates with other ancient concepts from the Near East.123 In Nephi’s account, it is a symbol of salvation, spiritual nourishment, and ascent into the presence of God, as well as a Messianic symbol, a symbol of the love of God, and a symbol of the mother of Christ. C. Wilfred Griggs discusses elements in the Book of Mormon account similar to Old World cultures, including:

(1) the difficulty of gaining access to it; (2) the various destructive forces around and about it; (3) the spiritual qualities required to make its fruit acceptable and nourishing to its partakers; and (4) the divine nature of the tree and its fruit, which render them unavailable to mortals lost in darkness and laden with sin.124

While Rick Grunder found an 1838 publication boasting of the abundant fruit in New York, nearly every state has fruit trees and regions that are well known for fruit (Washington apples, Georgia peaches, Wisconsin’s Door County for cherries, etc.). However, finding fruit in New York to explain the fruit of the tree of life is hardly interesting, and doesn’t address what really stands out in the Book of Mormon: the unique white fruit of the tree. This fruit is referenced later in the Book of Mormon, in text translated long before 1 Nephi was dictated, in Alma 32:40–42, where Alma has compared the word of God to a seed that can be planted in our heart and then grow, if carefully nourished, to yield “the fruit of the tree of life” (v. 40). Alma’s description of that fruit mirror’s Lehi’s, for it “is sweet above all that is sweet, and … white above all that is white, yea, and pure above all that is pure; and ye shall feast upon this fruit even until ye are filled, that ye hunger not, neither shall ye thirst” (Alma 32:42). 1 Nephi 8:11, Lehi explains that the tree’s fruit “was most sweet, above all that I ever before tasted. Yea, and I beheld that the fruit thereof was white, to exceed all the whiteness that I had ever seen.” Later, after Nephi has his version of the vision, he reports in that “the beauty [of the tree of life] was far beyond, yea, exceeding of all beauty; and the whiteness thereof did exceed the whiteness of the driven snow” (1 Nephi 11:8). This is not a New York apple tree.

[Page 200]Margaret Barker was impressed with Lehi’s description. She writes:

The tree of life made one happy, according to the Book of Proverbs (Proverbs 3:18), but for detailed descriptions of the tree we have to rely on the noncanonical texts. Enoch described it as perfumed, with fruit like grapes (1 Enoch 32:5), and a text discovered in Egypt in 1945 described the tree as beautiful, fiery, and with fruit like white grapes.125 I do not know of any other source that describes the fruit as white grapes. Imagine my surprise when I read the account of Lehi’s vision of the tree whose white fruit made one happy, and the interpretation that the Virgin in Nazareth was the mother of the Son of God after the manner of the flesh (1 Nephi 11:14–23).126 This is the Heavenly Mother, represented by the tree of life, and then Mary and her Son on earth. This revelation to Joseph Smith was the ancient Wisdom symbolism, intact, and almost certainly as it was known in 600 bce.127

Barker above touches upon the complex issue of the divine mother and the Asherah, the tree-like symbol from pre-reform Israelite religion, a topic also explored by Daniel Peterson, who finds significant evidence for early Near Eastern concepts in Nephi’s links between the tree of life, the Savior, and Mary.128 Brant Gardner, after reviewing scholarship on the role and meaning of the Asherah, observes:

The cultural linkage between the tree and Asherah explains how Lehi moved so easily from the Tree to the Messiah, and how Nephi so readily moved from the Tree to the “mother of God.” In pre-reform Israelite religion, Asherah was the divine mother. Therefore, Asherah as the Tree of Life was a logical symbol for the Messiah’s physical birth. The issue of whether Asherah should be considered a part of true Israelite belief is quite another discussion. The important information is that the complex of ideas surrounding Asherah would have informed Lehi and Nephi’s cultural understanding. That cultural background allowed them to easily make a transition from the tree to the mother of Yahweh, a transition difficult for us but natural for them.129

Related to the complex of ideas around the Asherah are the ancient Israelite traditions around Wisdom, personified as a woman in Proverbs 8 and in many other canonical and extracanonical sources from the ancient Near East. For example, speaking of wisdom, Psalm 3:18 says, [Page 201]“She is a tree of life to them that lay hold upon her: and happy is every one that retaineth her.” In the Book of Mormon, one must take hold of the iron rod (1 Nephi 8:24) or “lay hold” (Helaman 3:29) upon the word of God and upon arriving at the tree of life, as we learn in Lehi’s vision, one must persevere rather than wander into forbidden paths and be lost (1 Nephi 8:25, 28). It is staying at the tree of life and partaking of its fruit that brings happiness (1 Nephi 8:12; 11:21–23).

Peterson notes parallels between ancient Wisdom literature and concepts in Lehi’s dream and elsewhere in the Book of Mormon, including a potential wordplay.130 In related research, John Tvedtnes has found several early Christian texts in which Mary is associated with a tree or particularly the tree of life, as in 1 Nephi.131

Insights on a Temple Gone Dark: The Use of “Spacious” in the Book of Mormon

The Book of Mormon’s use of the term “spacious” is another interesting twist in this story. That word is not used in the King James Bible, but is consistently used in a negative context in the Book of Mormon. And in most cases, possibly all, it has an architectural connection (buildings). Thus we have “spacious buildings” (Mosiah 11:8–9), referring to Noah’s “elegant and spacious buildings” and “spacious palace,” and then Mormon’s condemnation of Riplakish, who taxed the people to “build many spacious buildings” in Ether 10:5. But before we read of the great and spacious building, Nephi introduces “spacious” to describe a field, of all things. But there’s something unusual about this field and the other words used to describe it in 1 Nephi 8, as Nephi quotes Lehi:

9 And it came to pass after I had prayed unto the Lord I beheld a large and spacious field.

20 And I also beheld a strait and narrow path, which came along by the rod of iron, even to the tree by which I stood; and it also led by the head of the fountain, unto a large and spacious field, as if it had been a world. [emphasis added]

A large and spacious field? As if it had been a world? This always sounded odd to me — until I read D. John Butler’s book, Plain and Precious Things: The Temple Religion of the Book of Mormon’s Visionary Men.132 Butler identifies numerous temple themes in Nephi’s writings, and explains how the three parts of the ancient Jewish temple are reflected there, as I previously mentioned in a 2012 post, “A Temple Gone Dark,”133 (written before I noted the use of “spacious” elsewhere in the [Page 202]Book of Mormon, which strengthens the argument made there). Among the three parts of the Jewish temple, first is the ulam, often translated as “porch,” a room that may be roofless or very tall. Then comes the hekal, the main middle room. That word literally means “building” or “great building.” A high, lofty building. And then comes the debir, the holy of holies, representing the presence and power of the Lord.134

As Lehi begins his travel in the dream, he encounters a “dark and dreary wilderness” that joins a “large and spacious field, as if it had been a world” (1 Nephi 8:20). The Hebrew word ulam for the first part of the temple is very close, almost identical in sound, to olam, the word that means “world.” In Butler’s view, there is a Hebrew play on words linking the great and spacious field, “a world,” to the temple’s ulam.135 If “the world” is a play on words linked to the courtyard of the temple, then “spacious” again could convey an architectural sense. There is a great and spacious courtyard, but dark and dreary from apostasy.

After the ulam comes the hekal, the “great building.” Recall Lehi’s words of what he saw after the spacious field/world/ulam, describing:

a great and spacious building; and it stood as it were in the air, high above the earth. And it was filled with people, both old and young, both male and female; and their manner of dress was exceedingly fine; and they were in the attitude of mocking and pointing their fingers towards those who had come at and were partaking of the fruit. (1 Nephi 8:26–27)

The word “fine” is used repeatedly in the Old Testament to describe the clothing of the priests in the temple, not secular clothing. The people with the fine clothing in the great and spacious building include the priests of the temple in a sinister hekal, part of Lehi’s dark temple experience.136 Butler also compares the fumes of incense that are part of the hekal with the mists of darkness that lead people astray.137 The waters of life that are part of many temple scenarios in ancient literature are replaced with dangerous filthy waters.

Only those who resist the corrupt religious establishment of Lehi’s day and the temptations and pressures of the adversary, clinging to the word of God (the iron rod) can make it past the dark ulam and sinister hekal and arrive safely to the debir and the tree of life, also rich in temple imagery.138

As is so often the case, there is much more going on in the Book of Mormon than meets the eyes of a casual reader rushing through the text.[Page 203]

A Plagiarized Straight and Narrow Path?

Some critics see evidence of plagiarism or modern origins in Nephi’s language about the “straight and narrow path.” First, I must say that I agree with John Welch’s very thoughtful and intelligent discussion of the confusion around “strait and narrow” versus “straight and narrow.”139 He argues convincingly that the word should be “straight” as it was printed in all editions of the Book of Mormon until 1981.

Whether “strait” or “straight,” the direct combination with “narrow” does not occur in the Bible, but does occur in Pilgrim’s Progress, a widely known Christian tome published by John Bunyan in 1678.140 In a dream, Goodwill tells the protagonist, Christian, that there are many ways that go down, “and they are crooked and wide; but thus thou mayest distinguish the right from the wrong, the right only being straight and narrow.” Did Joseph plagiarize from John Bunyan?

The phrase is actually older than Pilgrim’s Progress. The use of “straight” near “narrow” is, of course, found in Matthew 7:13–14:

Enter ye in at the strait gate: for wide is the gate, and broad is the way, that leadeth to destruction, and many there be which go in thereat: Because strait is the gate, and narrow is the way, which leadeth unto life, and few there be that find it.

Welch explains that “Had the Lord said, ‘Strait is the gate, and straight and narrow is the way,’ it would have been more descriptive but less poetic.”141 But there is really no need to specify the shape of the path in this bit of poetry. Crooked, winding paths are already ruled out in the scriptures His audience would have known (e.g., Deuteronomy 5:32–33; see also Psalms 5:8 which asks the Lord to “make thy way straight before my face” and Isaiah 40:3, “make straight in the desert a highway for our God”).

Bunyan was not the first to see that the Lord’s narrow path was also straight, not just strait. According to Welch:

Cyprian, a church father of the third century, in an apparent paraphrasing of Matthew 7:13–14, wrote, “How broad and spacious is the way which leadeth unto death, and many there are who go in thereby: how straight and narrow is the way that leadeth to life, and few there are that find it!” He also wrote, “We must persevere in the straight and narrow road of praise and glory.” [emphasis added]142

[Page 204]Origen also wrote of the “the straight and narrow way, which leads to life.”143

Welch attributes the popularity of the phrase to Bunyan’s influence. However, a search of Google Books shows it was also in use in modern English, or rather, Early Modern English, before Bunyan’s day, when the Early Modern English era was nearing its end.144 For example, the opening page of John Dee’s 1591 “Dr. Dee’s Apology” sent to the Archbishop of Canterbury speaks of the “true, straight, and most narrow path” of Christians.145 Two examples from 1632, both in a Christian context, including a work by Richard Hooker et al.146 and a work by Robert Chetwind,147 have “straight and narrow.” Examples are easier to find using a database of Early Modern English such as the Early English Books Online (EEBO) proximity search at the University of Michigan.148 There one can find, for example, a poem published by Robert Albott in 1600 with “For straight and narrow was the way that he did showe.”149 In 1608, Thomas Bell wrote, “First, that the way to heauen (that is to say Gods commaundements) is very straight and narrow, not wide and long, or easie.”150 There are other examples of “straight and narrow” in non-religious contexts, indicating that the pairing was more useful than just paraphrasing scripture. There are many dozens of examples to consider, with many obviously referring to the way to salvation.

One noteworthy point is that “straight and narrow” was not only part of English vocabulary in Joseph’s day but was also part of the vernacular of Early Modern English. I mention this because an important observation about the language of the Book of Mormon — not a theory that we Mormons need to buttress our faith but a fact-based observation that we are struggling to understand — is that much (not all) of the language of the Book of Mormon shows strong apparent influence from Early Modern English in ways that are not readily derived from the kjv Bible, almost as if there were some form of tight control in the translation to give an English text that was often moved away from the English of Joseph’s day or from kjv English into something slightly earlier and strangely different, yet plain and familiar, readily understandable to English speakers (unlike some Early Modern English). With this came grammar that is bad by modern standards but acceptable in Early Modern English, a story that has been well covered before.151 For now, the important thing is that “straight and narrow,” though related to the kjv, is not a direct kjv phrase, but was an established phrase before Bunyan came along. While its presence in the Book of Mormon may come from Joseph’s own vernacular, as we might expect with a translation, it is also consistent [Page 205]with the unexpected observation that there are many instances of text in the Book of Mormon showing Early Modern English influence.

One skeptic objected to the proposal that “straight and narrow” might be explained as part of Joseph’s vocabulary or as Early Modern English possibly transmitted to Joseph Smith through some form of tight control. “You can’t have it both ways!” he told me.152 However, I have it both ways all the time when I translate between Chinese and English, as do many others in translation work. I turn to automated tools or Chinese friends who give me words directly, but I may edit those myself or do translation in my own words at other times. Normal translation is a complex process and the Book of Mormon itself shows much complexity in the language used. If any mental effort was required from Joseph, and it appears that it was, then his mind and language were not entirely separated from the text. To require that every word in the Book of Mormon fit into a single model of translation or a single straightforward process, tight or loose, is unrealistic.153

Further, the charge of “plagiarism” is inappropriate. Using a well known phrase that has entered into the common vocabulary of a language is not plagiarism. Those who speak of quantitative easing, global warming, a black swan event, a utopian society, etc., are drawing upon recently developed phrases that can legitimately be used in an original work because they are part of our language now, as “straight and narrow” was in Joseph Smith’s day, and as it was in the Early Modern English era.

Whether the account of Lehi’s dream was dictated with tight control using an Early Modern English base text or “setting” of some kind, or whether it was translated more loosely in Joseph’s own vernacular, “straight and narrow” can be used to describe the path leading to eternal life even if that is not literally how the straightness or strictness of the way was expressed on the gold plates. It is a plausible term to use in a translation and is not a sign of “plagiarism.”

An Ancient Desert Landscape?

The landscape in Lehi’s dream and Nephi’s vision arguably fits the features of the Arabian Peninsula in ways that verdant Rochester does not. The river of filthy water has been compared to the dangerous, dirty, debris-laden flows that occur in Arabian wadis after a rainstorm.154 The broad paths that lead to destruction can be compared to the confusing landscape among desert wadis that channel in many directions and where death can be swift for those journeying in dangerous paths, [Page 206]especially if swept away by flash floods or, more commonly, faced with heat and lack of water. Further, as S. Kent Brown observes,

Lehi’s dream began in “a dark and dreary wilderness” wherein Lehi and a guide walked “in darkness” for “many hours” (1 Nephi 8:4, 8). Plainly, they were walking at night, the preferred time for traveling through the hot desert.155

Further, the great and spacious building rising as if it were in the air has been compared to buildings such as fortresses in the Arabian Peninsula rising from rocky outcroppings or buttes in the desert, which when lit up from within at night would appear to be floating in the air. However, the region most famous for lofty structures in Arabia, Yemen, would not have been encountered yet when Lehi had his vision relatively early in their journey, but viewing such structures later could have reinforced the image of the dream. S. Kent Brown writes:

The “great and spacious building” of Lehi’s dream appeared unusual enough to his eye that he called it “strange” (1 Nephi 8:33). He also wrote that this building in his dream “stood as it were in the air, high above the earth” (1 Nephi 8:26). Why would Lehi, who had evidently traveled a good deal during his life (he possessed “tents,” 1 Nephi 2:4), call a building strange? And does the word strange fit with the fact that the building soared into “the air, high above the earth”? Evidently, Lehi’s descriptions of this building point to architecture unfamiliar to him. Furthermore, his words prophetically anticipate architecture that he and his party would see in south Arabia.

Recent studies have shown that the so-called skyscraper architecture of modern Yemen, featured most vividly by the towering buildings in the town named Shibam in the Hadhramaut Valley, has been common since at least the eighth century bc and is apparently unique in the ancient world. The French excavations of the buildings at ancient Shabwah in the 1970s, including homes, indicate that the foundations of these buildings supported multistoried structures. In addition, “many ancient South Arabian building inscriptions indicate the number of floors within houses as three or four, with up to six in [the town of] Zafar.” Adding to the known details, “these inscriptions also provide the name of the owners” of these buildings.

[Page 207]In this light, it seems evident that Lehi was seeing the architecture of ancient south Arabia in his dream. For contemporary buildings there “stood as it were in the air,” rising to five or six stories in height. Such structures would naturally give the appearance of standing “high above the earth” (1 Nephi 8:26).156

Writing of Yemen’s architectural landscape, Yusuf ‘Abdullah writes:

The towns and villages of the country’s central region, bathed in sunshine from dawn till dusk, are situated in the steep mountains which tower into the blue sky. … Viewed from a distance, the houses and other buildings of these human settlements seem themselves like silent cliffs and hillocks that have grown out of the bedrock. … The dwellings atop the stony peaks and hills and in the valleys were built of carefully hewn stone from the local quarries. Most of them have several storeys and form settlements or villages capable of … defending the community.157

He also refers to the 4th-century historian and scholar, al-Hasan ibn Ahmad al-Hamdani, who described buildings and towns that are still reflected in the architecture of modern Yemen (see Figure 6). One fortress described by al-Hamdani was at Na’it, a white fortress on top of a mountain, while Sana’a was said to have a palace as tall as 20 stories, which may be exaggerated, but reflects ancient Yemen’s fascination with tall structures.158 Though al-Hamdani was long after Lehi, as Brown observed, there is evidence that the desert skyscrapers of Yemen have their roots in much more ancient times and may have been part of the landscape Lehi would see. However, such buildings are not mentioned by Nephi, weakening Brown’s proposed architectural significance of the great and spacious building.159

In light of the architectural, cultural, and geographical aspects of Lehi’s dream, Brown concludes that:

Lehi’s dream, perhaps more than any other segment of Nephi’s narrative, takes us into the ancient Near East. For as soon as we focus on certain aspects of Lehi’s dream, we find ourselves staring into the world of ancient Arabia. Lehi’s dream is not at home in Joseph Smith’s world but is at home in a world preserved both by archaeological remains and in the customs and manners of Arabia’s inhabitants. Moreover, from all appearances, the dream was prophetic — and I emphasize [Page 208]this aspect — for what the family would yet experience in Arabia. To be sure, the dream was highly symbolic. Yet it also corresponds in some of its prophetic dimensions to historical and geographical realities.160

Figure 6. Modern Shibam reflects the ancient Yemeni tradition of multistory buildings.161

Figure 6. Modern Shibam reflects the ancient Yemeni tradition of multistory buildings.161

Grunder, on the other hand, is convinced that the Book of Mormon wilderness is simply the verdant wilds and swamps of New York, since Nephi never mentions the desert, just “wilderness,” and since he feels thirst is not a significant problem for Lehi’s group as one would expect for a real journey through Arabia.162 However, the sufferings of Lehi’s group along Lehi’s trail did include thirst (Alma 18:37 and Alma 37:42) and did include many details consistent with a record from someone who had crossed Arabia as described.163 While Grunder thinks Nephi’s use of “wilderness” and his failure to use the word “desert” means Joseph was just thinking of the moist wilderness of New York when writing the Book of Mormon, 1 Nephi offers much more than anything Joseph could have dreamed up based on New England terrain. In dealing with a similar objection from RT, I point out that the word “wilderness” in the Book of Mormon is an appropriate translation for at least two commonly used biblical Hebrew terms that are sometimes also translated as “desert.”164 In fact, as the group came to the southern end of the Dead [Page 209]Sea, they would encounter the wide rift valley of Arabah, a name that actually means wilderness, just as Nephi had recorded. Nephi’s use of “wilderness” is reasonable and also subtly links their wilderness journey to the Exodus in a way entirely appropriate for an ancient Hebrew author describing a sacred journey to a new promised land.

The reality of Nephi’s trek through the desert in the Arabian Peninsula, not an imaginary jaunt through New York, is greatly underscored by archaeological evidence such as the ancient altars bearing a Nahom-related tribal name found in an appropriate region to validate an important part of Nephi’s record, and the discovery of an excellent candidate for Bountiful nearly due east of Nahom, just as Nephi wrote, issues that have been covered at length by Warren Aston and others.165 Grunder’s swift dismissal of the evidence is highly disappointing.

The growing body of recently discovered evidences related to Lehi’s Trail greatly strengthens the sense of authenticity of Nephi’s account and suggests that Grunder’s exclusive focus on modern parallels is a tragic case of stubbornly looking in the wrong place.

Extensions of the Iron Rod: An Active, Divisive Rod, or Even a Sword?

Criticism about the rod of iron as an anachronistic structure seems to draw upon our modern views of iron railings. We assume that the rod of iron is a nicely anchored, stationary railing made according to modern standards, nicely cemented into place with supports ever 30 or so centimeters. But the rod of iron in Isaiah 11:4 is used for smiting, a rather dynamic act, and when Mormon appears to refer to the iron rod and other themes from Lehi’s dream in Helaman 3:29–30, he urges us to “lay hold upon the word of God which is quick and powerful, which shall divide asunder all the cunning and the snares and the wiles of the devil, and lead the man of Christ in a strait [straight] and narrow course across the everlasting gulf of misery…” The word, which we must hold, is “quick” — alive, active — and can “divide” the artifices of the Adversary.166 This suggests motion, the kind of motion you might get from a rod that is being wielded by a divine agent. In leading us to salvation (or to the tree of life), perhaps its action is also more than merely a passive support. Perhaps the iron rod is pulling us or actively moving us in the right direction. It actively wrecks Satan’s deceitful artifices while bringing us, perhaps vigorously, to our goal.

The physics may seem questionable, but this is from a dream. We’re not sure what Lehi saw. But importing modern images into the dream [Page 210]and then declaring that the dream seems too modern may be rather fallacious.

The possibility of the rod playing an active, dynamic role is not just Mormon’s idea in Helaman 3. Nephi, in explaining the significance of the rod of iron to his brothers, states that “it was the word of God, and whoso would hearken unto the word of God and would hold fast unto it, they would never perish; neither could the temptations and the fiery darts of the adversary overpower them unto blindness, to lead them away to destruction” (1 Nephi 15:24). Thus, as Tvedtnes has noted, “This makes the rod both a source of support (as the word of God) and a weapon of defense against the devil’s ‘fiery darts’ ….”167 Nephi’s concept, nicely built into Helaman 3, suggests the role of the iron rod is more than just a static railing. Zachary Nelson has compared the iron rod in Lehi’s dream to tools used for defense and gathering such as the shepherd’s rod or staff, and rods in the Old Testament used as weapons to smite others.168 Nelson also notes that a rod can serve as a measuring stick (related perhaps to the concept of the scriptural canon) and as a scepter, again reminding us of its role as a symbol of authority.

If the rod Lehi saw was an exaggerated iron scepter, a symbol of God’s power and also of the word of God, building on the clever wordplay suggested by Matthew Bowen above, then in the dream it could have served as a barrier/railing but also as a dynamic tool to protect people and draw them home. Lehi doesn’t say it was permanently anchored, just that “it extended along the bank of the river and led to the tree” (1 Nephi 8:19). There was a path along the rod of iron (1 Nephi 18:20) and since a path is static, the rod may have been, but this is not necessary. The people who reached the tree of life “caught hold of the end of the rod of iron” and then pressed forward, “clinging to the rod or iron” (1 Nephi 8:24). It had a finite length, and the key was grabbing the end of it and holding on.169 That makes sense for a static structure, but it need not be, especially in a dream.

What if we compare the rod of iron with another metallic symbol of power, a sword? As we can see in Royal Skousen’s Analysis of Textual Variants of the Book of Mormon, 1 Nephi 12:18 currently has “the word of the justice of the Eternal God” serving to “divide” the wicked from the blessings of eternal life, but it should actually read “the sword of the justice of the Eternal God.”170 It is the sword, not the word, that is doing the dividing, which is more logical and consistent with ancient usage and with the dividing action in Helaman 3:29, though there it is [Page 211]the word of God carrying out that action. As Tvedtnes has pointed out, rods, swords, and the word of God may all be connected.171

Tvedtnes sees Helaman 3:29 as a build upon Lehi’s dream, but with the word likened to a sword, based on common language with Hebrews 4:12:

For the word of God is quick, and powerful, and sharper than any two-edged sword, piercing even to the dividing asunder of soul and spirit, and of the joints and marrow, and is a discerner of the thoughts and intents of the heart.

Tvedtnes writes:

The epistle to the Hebrews probably quoted a more ancient source, which was also borrowed in Helaman 3:29–30. …

The inclusion of the “strait and narrow course” and the “gulf of misery,” along with the “snares and the wiles of the devil,” clearly ties this passage to Lehi’s vision, where it is the rod or the word of God that brings people safely past Satan’s obstacles (the mist of darkness, the gulf, the fiery darts of the adversary, and the forbidden paths) to the tree of life (I Nephi 8:19–24; 12:17–18; 15:24,28). In the Helaman passage, however, the word of God seems to be compared to a sword.

The power of the word of God was emphasized by Alma; he noted that “it had had more powerful effect upon the minds of the people than the sword, or anything else, which had happened unto them” (Alma 31:5; cf. 61:14; Ecclesiastes 9:18). This reminds us that Nephi and other Book of Mormon prophets spoke with “the sharpness of the power of the word of God” (2 Nephi 1:26; Words of Mormon 1:17; Moroni 9:4; cf. Alma 1:7).172

The ability of the quick and powerful word of God to “divide asunder” in light of Hebrews 4:12 certainly suggests the action of a sword, but this may not necessarily be intended in the text and may be an artifact of the translation process that uses familiar kjv language, including numerous phrases, rather heavily. But does the similarity in phrasing in Helaman 3 really mean that the author intended to depict the word of God as a sword here? In fact, the word of God as a rod may still have been intended. “Laying hold” of a sword, especially Paul’s double-edged sword, can be a dangerous exercise, in contrast to the safety suggested by laying hold of the word. The ability of a sword to divide asunder is also within the [Page 212]scope of specifications for biblical rods. As Moses brings the Israelites out of Egypt, the Lord commands him to “lift up thy rod, and stretch out thine hand over the sea, and divide it: and the children of Israel shall go on dry ground through the midst of the sea” (Exodus 14:16). The gulf of death before the Israelites was safely crossed through the action of the rod of Moses and its power to divide the sea.

Regardless of the original language and intent in Helaman 3, the relationship between the sword and the word in the Book of Mormon squares nicely with recently recognized relationships in ancient Hebrew texts, as Joshua Berman demonstrated several years after Tvedtnes’ article on the rod, the sword, and the word.173 Berman explains that the term for double-edged or multi-edged sword, whether in Hebrew, Greek, or Aramaic, is literally a “sword of mouths” and typically refers figuratively to the power of speech.174 This further illustrates ancient connections between the sword and the word. Yuval Harari’s discussion of Jewish lore about the sword of Moses and its connections to the engraved name of God, the Torah, and the power of prayer may also be of interest.175

While swords and rods are symbols of power, protection, and smiting, I suggest that Helaman 3:29 is evoking both the image of the rod of Moses as well as the rod of Lehi’s dream, both associated with deliverance from a gulf and the quest to reach the promised land or tree of life. The word, once firmly gripped, can lead us to salvation, as Nephi’s iron rod which “led to the tree” (1 Nephi 8:19) and helps us ward off the fiery darts of the adversary (1 Nephi 15:24) that would otherwise destroy us or lead us to destruction.

Another connection occurs in 1 Nephi 15:30, when Nephi explains “that our father also saw that the justice of God did also divide the wicked from the righteous; and the brightness thereof was like unto the brightness of a flaming fire….” The bright, flaming justice of God, a sword (as originally in 1 Nephi 12:18) that divides or separates the wicked from the tree of life, here appears to draw upon the image of the cherubim and flaming sword of Genesis 3:24, placed there by God “to keep the way of the tree of life.” The “way” is derek, Strong’s H1870, which means road, path, etc.176 Lehi’s dream seems to build on that concept. The divine sword of justice/the sword of cherubim and a divine rod can all have a dividing effect. They can smite. They can be symbols of authority and power. And they seem to be used with skill and propriety in the Book of Mormon.

There may be more to the iron rod than meets the modern eye, and much more to the Book of Mormon’s use of that theme than Joseph could [Page 213]have gleaned from a quick glance at a Rochester aqueduct in his frantic final moments of dictating the beginning of the Book of Mormon. The alleged weakness of an anachronistic iron rod structure in Lehi’s dream may actually be a strength pointing to sophisticated usage drawing upon ancient concepts and even ancient Near Eastern wordplays.

Conclusion

The problem with looking only at modern sources to explain the Book of Mormon is that it leaves one blind to the abundant evidence of ancient origins. A fair evaluation should consider the Book of Mormon in the context it offers, determine if it is plausible, and weigh how the evidence for ancient origins compares to other theories.

A useful theory of Book of Mormon origins should explain how the text was generated. It should not just account for a few scattered elements, but should also be able to explain the strengths of the text, including new discoveries showing pervasive links to the ancient world and intricate craftsmanship in the text. Grunder’s theory does none of this.

There’s on old joke about a man on his hands and knees looking for something on the ground under a streetlight one night. A passerby asked him what he was doing. “Looking for a lost key.” Where did you lose it? “About a block down the street.” So why aren’t you searching there? “Because the light is better here.” Searching for parallels in the modern era is more convenient, but it’s not the right place to fairly evaluate the Book of Mormon.

As is often the case, when looking for parallels to a text in the wrong place, something can always be found, but what is found may not be as meaningful or informative as the parallels encountered when one searches nearer the source. The fake “keys” to the Book of Mormon from Joseph’s environment don’t really open the book to us. They don’t fit the data. Whether it’s a railing and a building in Rochester or a book like Pilgrim’s Progress as purported sources for a section of Nephi’s writing, they fail on numerous counts and don’t come close to offering plausibility or explanatory power for the riches that are there.

The ancient Old World connections related to the tree of life vision and the iron rod suggest that the Book of Mormon account is rooted in antiquity. Given the evidence from the text and external sources regarding the tree of life vision, Grunder’s theory of fabrication is far more fanciful, far more modern, and far less enduring than Nephi’s account.[Page 214]

Endnotes

1. Appendix 1: “The Rod of Iron: A Double in Dublin? Appendix for “The Great and Spacious Book of Mormon Arcade Game,” JeffLindsay.com, Oct. 5, 2016, http://www.jefflindsay.com/lds/rod-of-iron-in-dublin/. Appendix 2: “Walt Whitman as the Source for Lehi’s Dream and More?,” JeffLindsay.com, Oct. 5, 2016, http://www.jefflindsay.com/lds/walt-whitman-lehis-dream/.

2. “The Spalding Theory of Book of Mormon Authorship,” FAIRMormon, http://en.fairmormon.org/Book_of_Mormon/Authorship_theories/Spalding_manuscript, accessed June 9, 2016.

3. Jeff Lindsay, “Joseph and the Amazing Technicolor Dream Map: Part 1 of 2,” Interpreter: A Journal of Mormon Scripture 19 (2016): 153–239, https://journal.interpreterfoundation.org/joseph-and-the-amazing-technicolor-dream-map-part-1-of-2/, and Jeff Lindsay, “Joseph and the Amazing Technicolor Dream Map: Part 2 of 2,” Interpreter: A Journal of Mormon Scripture 19 (2016): 247–326, https://journal.interpreterfoundation.org/joseph-and-the-amazing-technicolor-dream-map-part-2-of-2/.

4. RT, “Critiques of Nahom and Lehi’s Journey,” Faith-Promoting Rumor, May 9, 2016, http://www.patheos.com/blogs/faithpromotingrumor/2016/05/critiques-of-nahom-and-lehis-
journey/, accessed May 10, 2016.

5. Jeff Lindsay, “The Great and Spacious Book of Mormon Arcade Game,” Mormanity Blog, May 23, 2016, http://mormanity.blogspot.com/2016/05/the-great-and-spacious-book-of-mormon.html.

6. Rick Grunder, “The Dream of the Iron Rod,” PDF file taken from Entry 350, “Reynolds Arcade (Rochester, New York),” in Mormon Parallels: A Bibliographic Source, 2nd ed. (Lafayette, NY: Rick Grunder — Books, 2014), 1367–1431; available at http://www.rickgrunder.com/parallels/mp350.pdf.

7. RT, “Critiques of Nahom and Lehi’s Journey.”

8. RT, “Nahom and Lehi’s Journey through Arabia: A Historical Perspective, Part 3” (hereafter Part 3), Faith Promoting Rumor, Patheos.com, Oct. 24, 2015, http://www.patheos.com/blogs/faithpromotingrumor/2015/10/nahom-and-lehis-journey-through-arabia-a-historical-perspective-part-3/, here quoting Philip Jenkins, “The Nahom Follies,” The Anxious Bench, Patheos.com, June 14, 2015; http://www.patheos.com/blogs/anxiousbench/2015/06/the-nahom-follies/.

9. Benjamin L. McGuire, “Finding Parallels: Some Cautions and Criticisms, Part One,” Interpreter: A Journal of Mormon Scripture 5 (2013): 1–59, https://journal.interpreterfoundation.org/finding-parallels-some-cautions-and-criticisms-part-one/ and “Finding Parallels: Some Cautions and Criticisms, Part Two,” Interpreter: A Journal of Mormon Scripture 5 (2013): 61–104, https://journal.interpreterfoundation.org/finding-parallels-some-cautions-and-criticisms-part-two/.

10. Rick Grunder, “The Great and Spacious Building,” Worlds Without End, April 27, 2015, http://www.withoutend.org/great-spacious-building/.

11. Grunder, “The Dream of the Iron Rod.”

12. “Old Rochester Aqueduct, Genesee River, Rochester, NY,” The Travels of Tug 44, http://www.tug44.org/canal.history/rochester-aqueduct/, accessed June 17, 2016.

13. Frank E. Sadowski, Jr., “Images of the Erie Canal in Rochester and Vicinity,” ErieCanal.org, http://eriecanal.org/Rochester-1.html, accessed June 17, 2016.

14. Frank E. Sadowski, Jr., “The Broad Street Bridge,” section in “Contemporary Photographs from Rochester,” ErieCanal.org, http://eriecanal.org/Rochester-2.html, accessed June 17, 2016.

15. Lithograph from the cover of William C. Glynn, “The Rochester Arcade Quick Step,” (1844), Historic Sheet Music Collection, Paper 651, ConnColl.edu, http://digitalcommons.conncoll.edu/sheetmusic/651, with the PDF file at http://digitalcommons.conncoll.edu/cgi/viewcontent.cgi?article=1650&context=sheetmusic, accessed Aug. 4, 2016. Also see Diane Shaw, City Building on the Eastern Frontier: Sorting the New Nineteenth Century City (Baltimore: Johns Hopkins University Press, 2004), 124–130 (much of the text is viewable at https://books.google.com/books?id=c9m5lKqOUecC&&pg=PA123).

16. Elisha Johnson, A Directory for the Village of Rochester (Rochester, NY: Elisha Ely and Everard Peck, 1827), as cited by Grunder, “The Dream of the Iron Rod.”

17. “View of Rochester with a section of the Aqueduct”, 1830, wood engraving from “Monthly Repository and Library of Entertaining Knowledge,” 1/5 (October 1830), provided at “Images of the Erie Canal in Rochester and Vicinity,” ErieCanal.org, http://eriecanal.org/Rochester-1.html, accessed Aug. 3, 2016.

18. Shaw, City Building on the Eastern Frontier, 125.

19. Ibid., 123.

20. RT, “Critiques of Nahom and Lehi’s Journey.”

21. Grunder, “The Dream of the Iron Rod,” 1383.

22. Ibid., 1371.

23. RT, “Critiques of Nahom and Lehi’s Journey,” 2016.

24. Recommended readings include: (1) Bob Marcotte, “Reynolds Arcade,” from the “Retrofitting Rochester” series in partnership with the Office of the City Historian of Rochester, Democrat and Chronicle, Rochester, NY, 2012, http://media.democratandchronicle.com/retrofitting-rochester/reynolds-arcade; (2) “Walking Tour of Rochester’s One Hundred Acre Plot,” LowerFalls.org, http://www.lowerfalls.org/100acrestour/, which features several photos and drawings of the Reynolds Arcade and other prominent buildings in Rochester, with some history; (3) “Reynolds Arcade,” Libraryweb.org, Monroe County (NY) Library System, http://www.libraryweb.org/rochimag/architecture/LostRochester/Reynolds/Reynolds.htm, with several historic views of the Reynolds Arcade; and (4) Diane Shaw, City Building on the Eastern Frontier: Sorting the New Nineteenth Century City (Baltimore: Johns Hopkins University Press, 2004), 124–130 (viewable at Amazon.com). Incidentally, Shaw points out that the glass roofing shown in some photos is from a remodeling effort long after Joseph might have seen the building.

25. Grunder, “The Dream of the Iron Rod,” 1402.

26. “History of RIT,” RIT.edu, https://www.rit.edu/overview/history-rit, accessed May 28, 2016.

27. Catalogue of the Rochester City Library, April 1839, provided by the Rochester Athenaeum and Young Men’s Association (Rochester, NY: Shephard, Strong, and Dawson, 1839), http://www.libraryweb.org/~digitized/books/Catalogue_of_the_Roch_City_Library_1839.pdf. Thanks to Mormanity reader James Anglin for finding this resource, as discussed in comments of Jeff Lindsay, “The Great and Spacious Book of Mormon Arcade Game,” Mormanity Blog, May 23, 2016, http://mormanity.blogspot.com/2016/05/the-great-and-spacious-book-of-mormon.html.

28. Josiah Conder, Arabia, in The Modern Traveler: A Popular Description, Geographical, Historical, and Topographical, of the Various Countries of the Globe, (London: James Duncan, 1825). The Modern Traveler and its 10 volumes are listed (without author, date, or publisher information) in Catalogue of the Rochester City Library, 17.

29. I have found four online editions of Conder’s books printed at three different dates, with all but the first lacking the full map of Arabia in the electronic document:

(1) a scan of the 1825 printing at ExploreDoc.com: Josiah Conder, Arabia, in The Modern Traveler: A Popular Description, Geographical, Historical, and Topographical, of the Various Countries of the Globe, (London: James Duncan, 1825), at “James Duncan, The Modern Traveller, Arabia, 1825,” Exploredoc.com, http://exploredoc.com/doc/3937504/james-duncan–the-modern-traveller–arabia–1825, which features the unfolded map of this volume before the title page;

(2) a c.1825 printing at Google Books: Josiah Conder, A Popular Description of Arabia: Geographical, Historical, and Topographical (London: James Duncan, 1825) at https://books.google.com/books?id=hugji007ouMC&pg=PA1 (no date is listed in the book but Google’s “About this book” info states it is from 1825);

(3) an 1830 printing at Archive.org: Josiah Conder, Arabia, vol. 4 of The Modern Traveler: A Description, Geographical, Historical, and Topographical, of the Various Countries of the Globe, in Thirty Volumes (London: James Duncan, 1830), https://archive.org/stream/moderntraveller04condiala#page/n3/mode/2up; and

(4) an 1833 version at Hathitrust.org: Josiah Conder, Arabia, Comprising Its Geography, History, and Topography (Philadelphia, PA: Thomas Wardle, 1833), https://babel.hathitrust.org/cgi/pt?id=loc.ark:/13960/t6xw53h33;view=1up;seq=17. All links accessed July 30, 2016. The fold-out map can also be seen in a display from Rooke Books of Bath, England, RookeBooks.com, http://www.rookebooks.com/product?prod_id=16526, accessed July 28, 2016. A larger image of Conder’s 1825 map can be viewed at “Topography; The Modern Traveller-Arabia-1825,” Catawiki.com, https://auction.catawiki.com/kavels/2005491-topography-the-modern-traveller-arabia-1825, accessed July 30, 2016. A clearer map, though black and white only, is available at the Exploredoc.com link given above.

30. The catalog number for Conder’s volume on Arabia is 1171. Nearby catalog numbers include 1191 for Grenville Mellen’s Book of the United States, which was printed in 1838 (see https://www.amazon.ca/book-United-States-Grenville-Mellen/dp/B006BYJ6D6/159-0006769-5803450?ie=UTF8&*Version*=1&*entries*=0#reader_B006BYJ6D6), and 1118, G.B. Cheever, An American Commonplace Book of Prose, printed in 1839 (see https://www.amazon.com/American-Common-Place-Book-Prose/dp/B002NSN21G), while titles mentioning earlier post-1830 dates are found in lower numbers such as 972, (Sketches of Travel in Turkey and Greece in 1831–2) and 796, Continent in 1835.

31. Conder, Arabia in Modern Traveller, 1825, fold out map viewable at Exploredoc.com, http://exploredoc.com/doc/3937504/james-duncan–the-modern-traveller–arabia–1825.

32. Conder, A Popular Description of Arabia, c.1825, RookeBooks.com, http://www.rookebooks.com/product?prod_id=16526. See also the map at Catawiki.com, https://auction.catawiki.com/kavels/2005491-topography-the-modern-traveller-arabia-1825.

33. Personal correspondence with Rooke Books, Bath, England, May 26, 2016.

34. Lindsay, “Joseph and the Amazing Technicolor Dream Map: Part 1.”

35. Stephen D. Ricks, “On Lehi’s Trail: Nahom, Ishmael’s Burial Place,” Journal of the Book of Mormon and Other Restoration Scripture 20/1 (2011): 66–68, https://scholarsarchive.byu.edu/cgi/viewcontent.cgi?article=1475&context=jbms.

36. Carsten Niebuhr, Travels Through Arabia and Other Countries in the East, 2 vols., trans. Robert Heron (Edinburgh: R. Morison & Son, 1792), vol. 1, 403, https://archive.org/stream/travelsthroughar11nieb#page/402/mode/2up. The relevant page has a misprinted page number, 340, though it should be 403.

37. Conder, Arabia, vol. 4 of Modern Traveller (1830), 340, https://archive.org/stream/moderntraveller04condiala#page/340/mode/2up; see also https://books.google.com/books?id=hugji007ouMC&pg=PA340 and https://babel.hathitrust.org/cgi/pt?id=loc.ark:/13960/t6xw53h33;view=1up;seq=352.

38. Strong’s H5098, Blue Letter Bible, https://www.blueletterbible.org/lang/Lexicon/Lexicon.cfm?strongs=H5098&t=kjv. See especially the entry from Gesenius’ Hebrew-Chaldee Lexicon. Also see the entry for naham in Roy, A Complete Hebrew and English Critical and Pronouncing Dictionary, 499, https://books.google.com/books?id=SuE5AQAAMAAJ&pg=PA499.

39. “Rochester Athenaeum Collection,” RIT.edu, http://library.rit.edu/findingaids/html/RITArc.0028.html, accessed June 5, 2016.

40. Michael Hubbard MacKay and Gerrit J. Dirkmaat, From Darkness Unto Light: Joseph Smith’s Translation and Publication of the Book of Mormon (Provo, UT: Religious Studies Center, BYU and Salt Lake City: Deseret Book, 2015), 141–42.

41. Robert Paul, “Joseph Smith and the Manchester (New York) Library,” BYU Studies 22/3 (Summer 1982): 333–356, https://byustudies.byu.edu/content/joseph-smith-and-manchester-new-york-library.

42. Kansas City Daily Journal, 5 June 1881, as quoted at FAIRMormon, http://en.fairmormon.org/Question:_What_is_the_date_of_the_restoration_of_the_Melchizedek_Priesthood%3F#cite_note-4.

43. John W. Welch, “The Miraculous Translation of the Book of Mormon,” Opening the Heavens: Accounts of Divine Manifestations, 1820–1844, ed. John W. Welch, (Salt Lake City: Deseret Book; Provo, UT: Brigham Young University Press, 2005), 76–213, citation at 80, https://byustudies.byu.edu.

44. MacKay and Dirkmaat, From Darkness Unto Light, 164.

45. Title Page, 1830 Book of Mormon, JosephSmithPapers.org, http://www.josephsmithpapers.org/paperSummary/book-of-mormon-1830, accessed Aug. 4, 2016. (The user must click on the right arrow several times to bring up the Title Page.)

46. John W. Welch, “How long did it take Joseph Smith to translate the Book of Mormon?,” Ensign 18 (Jan. 1988), 46, https://churchofjesuschrist.org/ensign/1988/01/i-have-a-question?lang=eng.

47. Grunder, “The Dream of the Iron Rod,” 1407.

48. MacKay and Dirkmaat, From Darkness Unto Light, 165.

49. See John H. Gilbert’s typescript memoir, “Recollections of John H. Gilbert [by himself],” 1892, archived at BYU, http://www.boap.org/LDS/Early-Saints/JHGilbert.html, accessed June 5, 2016.

50. Wayne Sentinel, June 26, 1829, as cited by MacKay and Dirkmaat, From Darkness unto Light, 165.

51. Pomeroy Tucker, Origin, Rise, and Progress of Mormonism: Biography of Its Founders and History of Its Church (New York: D. Appleton, 1867), 50–51, http://www.solomonspalding.com/docs1/1867TucA.htm.

52. Tucker, Origin, Rise, and Progress of Mormonism, 52.

53. For example, the chronology for Oliver Cowdery at OliverCowdery.com puts the trip at mid-June): “Oliver Cowdery Chronology,” OliverCowdery.com, http://olivercowdery.com/history/Cdychrn1.htm. Royal Skousen’s chronology for the Book of Mormon also shows the Rochester visit in “early” June 1829, listed before the ca. June 5–8 baptism of Hyrum Smith and David Whitmer. See “Book of Mormon Chronology for the Years 1823–1830” in The Joseph Smith Papers, Revelations and Translations, Volume 3: Part 2, Printer’s Manuscript of the Book of Mormon, Alma 36–Moroni 10, ed. Royal Skousen and Robin Scott Jensen (Salt Lake City: The Church Historian’s Press, 2015), 414.

54. MacKay and Dirkmaat, From Darkness Unto Light, 168.

55. Ibid, 175.

56. Gerrit J. Dirkmaat and Michael Hubbard MacKay, “Joseph Smith’s Negotiations to Publish the Book of Mormon,” in The Coming Forth of the Book of Mormon: A Marvelous Work and a Wonder, ed. Dennis L. Largey, Andrew H. Hedges, John Hilton III, and Jerry Hull, (Provo, UT: Religious Studies Center, Brigham Young University, 2015), 155–171, citation from 167.

57. Welch, “The Miraculous Translation of the Book of Mormon,” 98.

58. “Book of Mormon/Translation/Chronology,” FAIRMormon, http://en.fairmormon.org/Book_of_Mormon/Translation/Chronology, accessed Aug. 4, 2016.

59. Eldon Watson, “Approximate Book of Mormon Translation Timeline,” April 1995; http://www.eldenwatson.net/BoM.htm, accessed Aug 4, 2016.

60. The lost 116 pages are likely to have included details about Lehi’s travels and experiences that today could provide extensive information to validate the record as impossible to forge. As one anonymous peer reviewer of this paper noted, “one of the great ironies of the story of the lost 116 pages is that Harris’s drive to prove that he was not being deceived seems to have had the effect of eliminating the portion of the book that was most likely to result in his vindication on evidentiary grounds. His own desire for tangible proof caused that proof to elude him and to some extent us as well.” Personal communication, Sept. 29, 2016.

61. Walter Brueggemann, „From Dust to Kingship,“ Zeitschrift für die alttestamentliche Wissenschaft, 84/1 (1972): 1–18.

62. David Bokovoy, “Deutero-Isaiah in the Book of Mormon: A Literary Analysis (pt. 1),” When Gods Were Men, Patheos.com, April 29, 2014, http://www.patheos.com/blogs/davidbokovoy/2014/04/deutero-isaiah-in-the-book-of-mormon-a-literary-analysis-pt-1/.

63. Jeff Lindsay, “‘Arise from the Dust’: Insights from Dust-Related Themes in the Book of Mormon, Part 2: Enthronement, Resurrection, and Other Ancient Motifs from the ‘Voice from the Dust,’” Interpreter: A Journal of Mormon Scripture 22 (2016): 233-277, https://journal.interpreterfoundation.org/arise-from-the-dust-insights-from-dust-related-themes-in-the-book-of-mormon-part-2-enthronement-resurrection-and-other-ancient-motifs-from-the-voice-from-the-dust/.

64. Matthew L. Bowen, “‘See That Ye Are Not Lifted Up’: The Name Zoram and Its Paronomastic Pejoration,” Interpreter: A Journal of Mormon Scripture 19 (2016): 109–143, https://journal.interpreterfoundation.org/see-that-ye-are-not-lifted-up-the-name-zoram-and-its-paronomastic-pejoration/.

65. John A. Tvedtnes, “Reflections on Nephi’s Vision in His Closing Chapters,” Book of Mormon Research, Sept. 5, 2013; http://bookofmormonresearch.org/index/book_of_mormon_articles/reflections-nephis-vision.

66. Matthew L. Bowen, “Alma — Young Man, Hidden Prophet,” Interpreter: A Journal of Mormon Scripture 19 (2016): 343–353, https://journal.interpreterfoundation.org/alma-young-man-hidden-prophet/#more-8608.

67. See Paul Y. Hoskisson, “What’s in a Name? Alma as a Hebrew Name,” Journal of Book of Mormon Studies 7/1 (1998): 72–73, https://scholarsarchive.byu.edu/cgi/viewcontent.cgi?article=1192&context=jbms. See also John Tvedtnes, “Hebrew Names in the Book of Mormon,” Thirteenth World Congress of Jewish Studies, Jerusalem, 2001, available at http://www.fairmormon.org/wp-content/uploads/2011/12/tvedtnes-HebrewNames.pdf and John A. Tvedtnes, John Gee, and Matthew Roper, “Book of Mormon Names Attested in Ancient Hebrew Inscriptions,” Journal of Book of Mormon Studies 9/1 (2000) 40–51, 78–79, https://scholarsarchive.byu.edu/cgi/viewcontent.cgi?article=1239&context=jbms

68. Bowen, “Alma — Young Man, Hidden Prophet.”

69. Loren Blake Spendlove and Tina Spendlove, “Turning to the Lord With the Whole Heart: The Doctrine of Repentance in the Bible and the Book of Mormon,” Interpreter: A Journal of Mormon Scripture 20 (2016): 177–246, quotation at 212–3, https://journal.interpreterfoundation.org/turning-to-the-lord-with-the-whole-heart-the-doctrine-of-repentance-in-the-bible-and-the-book-of-mormon/.

70. John W. Welch, “Chiasmus in the Book of Mormon,” New Era, 2 (Feb. 1972): 6–11, https://churchofjesuschrist.org/new-era/1972/02/chiasmus-in-the-book-of-mormon?lang=eng, accessed June 28, 2016.

71. Richard Elliott Friedman, Who Wrote the Bible, 2nd edition (New York: HarperCollins, 1997). See also “Historical Criticism, Wikipedia, https://en.wikipedia.org/wiki/Historical_criticism, accessed Feb. 25, 2016.

72. Robert B. Chisholm Jr., “Old Testament Source Criticism: Some Methodological Miscues,” in Do Historical Matters Matter to Faith? A Critical Appraisal of Modern and Postmodern Approaches to Scripture, ed. James K. Hoffmeier and Dennis R. Magary (Wheaton, IL: Crossway, 2014), 181–199.

73. G.J. Wenham, “The Coherence of the Flood Narrative,” Vetus Testamentum 28 (1978): 336–48. A different chiastic structure was proposed by F.I. Andersen, The Sentence in Biblical Hebrew (The Hague, Netherlands: Mouton Publishing, 1974), 39–40, 59. The latter, according to David Bokovoy, is rebutted in J.A. Emerton, “An Examination of Some Attempts to Defend the Unity of the Flood Narrative in Genesis: Part II,” Vetus Testamentum 38 (1988): 1–21, and the very different chiastic structures proposed by Wenham and Andersen suggests to Bokovoy that the alleged chiastic structures are subjective. See David E. Bokovoy, Authoring the Old Testament: Genesis-Deuteronomy (Salt Lake City, UT: Greg Kofford Books, 2014), Kindle edition, Chapter Two, “Documentary Sources in the Pentateuch,” subsection “Challenges to the Theory.”

74. Welch, “Chiasmus in the Book of Mormon.”

75. Ibid.

76. Noel Reynolds, “Nephi’s Outline,” in Book of Mormon Authorship: New Light on Ancient Origins, ed. Noel B. Reynolds (Provo, UT: BYU Religious Studies Center, 1982; reprint FARMS, 1996), 53–74, https://archive.bookofmormoncentral.org/content/nephis-outline, accessed June 20, 2016.

77. Ibid., 56.

78. Ibid., 58–59.

79. Ibid., 59.

80. Ibid., 71–72.

81. Dennis Newton, “Nephi’s Use of Inverted Parallels,” Interpreter: A Journal of Mormon Scripture 22 (2016): 79–106, https://journal.interpreterfoundation.org/nephis-use-of-inverted-parallels/.

82. Matthew Nickerson, “Nephi’s Psalm: 2 Nephi 4:16–35 in Light of Form-Critical Analysis,” Journal of Book of Mormon Studies 6/22 (1997): 26–42, at 30, https://scholarsarchive.byu.edu/cgi/viewcontent.cgi?article=1165&context=jbms.

83. Steven P. Sondrup, “The Psalm of Nephi: A Lyric Reading,” BYU Studies, 21/3 (1981): 1–16, https://byustudies.byu.edu/content/psalm-nephi-lyric-reading.

84. “Is ‘Nephi’s Psalm’ Really a Psalm?,” Book of Mormon Central, February 10, 2016, https://knowhy.bookofmormoncentral.org/content/is-nephis-psalm-really-a-psalm, accessed July 7, 2016.

85. For an introduction to biblical wisdom literature, see Roland E. Murphy, The Tree of Life: An Exploration of Biblical Wisdom Literature, 3rd ed. (Grand Rapids, MI: Eerdmans, 2002). For issues related to the Book of Mormon and Book of Moses, see Samuel Zinner, “’Zion’ and ‘Jerusalem’ as Lady Wisdom in Moses 7 and Nephi’s Tree of Life Vision,” Interpreter: A Journal of Mormon Scripture 12 (2014): 281–323, https://journal.interpreterfoundation.org/zion-and-jerusalem-as-lady-wisdom-in-moses-7-and-nephis-tree-of-life-vision/.

86. Daniel C. Peterson, “Nephi and His Asherah,” Journal of Book of Mormon Studies 9/2 (2000): 16–25, 80–81, https://scholarsarchive.byu.edu/cgi/viewcontent.cgi?article=1253&context=jbms. Also see “Lady Wisdom,” chapter 9 in Murphy, The Tree of Life, 133–147.

87. “Why Does Nephi Always Go Down to the Wilderness and Up to Jerusalem?,” Book of Mormon Central, Jan. 7, 2016, http://www.knowhy.bookofmormoncentral.org/content/why-does-nephi-always-go-down-wilderness-and-jerusalem.

88. Paul Y. Hoskisson, “Was Joseph Smith Smarter Than the Average Fourth Year Hebrew Student? Finding a Restoration-Significant Hebraism in Book of Mormon Isaiah,” Interpreter: A Journal of Mormon Scripture 17 (2016): 151–158, https://journal.interpreterfoundation.org/was-joseph-smith-smarter-than-the-average-fourth-year-hebrew-student-finding-a-restoration-significant-hebraism-in-book-of-mormon-isaiah/.

89. Neal Rappleye, “Nephi the Good: A Commentary on 1 Nephi 1:1–3,” Interpreter Blog, January 3, 2014, https://journal.interpreterfoundation.org/nephi-the-good-a-commentary-on-1-nephi-11-3/, accessed July 10, 2016.

90. “Nahom,” Book of Mormon Onomasticon (Provo, UT: Brigham Young University), https://onoma.lib.byu.edu/index.php/NAHOM, accessed July 8, 2016.

91. Matthew L. Bowen, “Onomastic Wordplay on Joseph and Benjamin and Gezera Shawa in the Book of Mormon,” Interpreter: A Journal of Mormon Scripture 18 (2016): 255–273, https://journal.interpreterfoundation.org/onomastic-wordplay-on-joseph-and-benjamin-and-gezera-shawa-in-the-book-of-mormon/.

90. Matthew L. Bowen, “’Most Desirable Above All Things’: Onomastic Play on Mary and Mormon in the Book of Mormon,” Interpreter: A Journal of Mormon Scripture 13 (2015): 27–61, https://journal.interpreterfoundation.org/most-desirable-above-all-things-onomastic-play-on-mary-and-mormon-in-the-book-of-mormon/.

93. Matthew L. Bowen, “And There Wrestled a Man with Him (Genesis 32:24): Enos’s Adaptations of the Onomastic Wordplay of Genesis,” Interpreter: A Journal of Mormon Scripture 10 (2014): 151–160, https://journal.interpreterfoundation.org/and-there-wrestled-a-man-with-him-genesis-3224-enoss-adaptations-of-the-onomastic-wordplay-of-genesis/.

94. S. Kent Brown, “The Exodus Pattern in the Book of Mormon,” in From Jerusalem to Zarahemla: Literary and Historical Studies of the Book of Mormon (Provo, UT: Religious Studies Center, Brigham Young University, 1998), 75–98, https://rsc.byu.edu/archived/jerusalem-zarahemla-literary-and-historical-studies-book-mormon/exodus-pattern-book-mormon.

95. Ben McGuire, “Nephi and Goliath: A Reappraisal of the Use of the Old Testament in First Nephi,” 2001 FAIRMormon Conference, Salt Lake City, Aug. 2001, http://www.fairmormon.org/perspectives/fair-conferences/2001-fair-conference/2001-nephi-and-goliath-a-reappraisal-of-the-use-of-the-old-testament-in-first-nephi.

96. Dennis Newton, “Nephi’s Change of Heart,” Interpreter: A Journal of Mormon Scripture 20 (2016): 261–291, https://journal.interpreterfoundation.org/nephis-change-of-heart/. See also Newton, “Nephi’s Use of Inverted Parallels.”

97. Grant Hardy, Understanding the Book of Mormon (Oxford University Press, 2010), 11–25, 62–65, 84 (the “unique voice” of Nephi is mentioned particularly at 84). See also Daniel Peterson’s review in Daniel C. Peterson, “An Apologetically Important Nonapologetic Book,” Journal of Book of Mormon Studies 16 (2016): 52–75, https://scholarsarchive.byu.edu/cgi/viewcontent.cgi?article=1590&context=jbms (subscription required).

98. Margaret Barker, “What Did King Josiah Reform?” in Glimpses of Lehi’s Jerusalem, ed. Jo Ann H. Seely, David Rolph Seely, and John W. Welch (Provo, UT: Foundation for Ancient Research and Mormon Studies, Brigham Young University, 2004) 538, https://scholarsarchive.byu.edu/cgi/viewcontent.cgi?filename=18&article=1038&context=mi&type=additional. See also Neal Rappleye, “The Deuteronomist Reforms and Lehi’s Family Dynamics: A Social Context for the Rebellions of Laman and Lemuel,” Interpreter: A Journal of Mormon Scripture 16 (2015): 87–99, https://journal.interpreterfoundation.org/the-deuteronomist-reforms-and-lehis-family-dynamics-a-social-context-for-the-rebellions-of-laman-and-lemuel/.

99. Margaret Barker, “Joseph Smith and Preexilic Israelite Religion,” in The Worlds of Joseph Smith, ed. John W. Welch (Provo, UT: Brigham Young University Press, 2005), 69–82, https://ojs.lib.byu.edu/spc/index.php/BYUStudies/article/viewFile/7066/6715%22%5Dhere. Also see Kevin Christensen, “The Temple, the Monarchy, and Wisdom: Lehi’s World and the Scholarship of Margaret Barker,” in Glimpses of Lehi’s Jerusalem, ed. Jo Ann H. Seely, David Rolph Seely, and John W. Welch (Provo, UT: Foundation for Ancient Research and Mormon Studies, Brigham Young University, 2004), 449–522, https://scholarsarchive.byu.edu/cgi/viewcontent.cgi?filename=17&article=1038&context=mi&type=additional; Margaret Barker, Temple Mysticism: An Introduction (London: Society for Promoting Christian Knowledge, 2011; Kevin Christensen, “Book Review: Temple Mysticism: An Introduction, by Margaret Barker,” Interpreter: A Journal of Mormon Scripture, 5 (2013): 191–199, https://journal.interpreterfoundation.org/book-review-temple-mysticism-an-introduction-by-margaret-barker/; and Neal Rappleye, “The Deuteronomist Reforms and Lehi’s Family Dynamics: A Social Context for the Rebellions of Laman and Lemuel,” Interpreter: A Journal of Mormon Scripture 16 (2015): 87–99, https://journal.interpreterfoundation.org/the-deuteronomist-reforms-and-lehis-family-dynamics-a-social-context-for-the-rebellions-of-laman-and-lemuel/.

100. William J. Hamblin, “Reformed Egyptian,” FARMS Review 19/1 (2007): 31–35, https://scholarsarchive.byu.edu/cgi/viewcontent.cgi?article=1694&context=msr. See also William Hamblin, “Palestinian Hieratic,” Interpreter Blog, Sept. 1, 2012, https://interpreterfoundation.org/news-palestinian-hieratic/ and Neal Rappleye, “Learning Nephi’s Language: Creating a Context for 1 Nephi 1:2,” Interpreter: A Journal of Mormon Scripture 16 (2015): 151–159, https://journal.interpreterfoundation.org/learning-nephis-language-creating-a-context-for-1-nephi-12/.

101. William J. Hamblin, “Sacred Writing on Metal Plates in the Ancient Mediterranean,” FARMS Review, 19/1 (2007): 37–54, https://scholarsarchive.byu.edu/cgi/viewcontent.cgi?article=1695&context=msr. See also H. Curtis Wright in “Metal Documents in Stone Boxes,” in By Study and Also by Faith, ed. John M. Lundquist and Stephen R. Ricks, Vol. 1 (Salt Lake City: Deseret Book, 1990), 273–334; and Jeff Lindsay, “Hiding Sacred Records like the Golden Plates: A Well Established Ancient Practice,” JeffLindsay.com, http://www.jefflindsay.com/bme10.shtml.

102. John A. Tvedtnes, “’The Hilt Thereof Was of Pure Gold,’” Interpreter Blog, Oct. 4, 2015, https://interpreterfoundation.org/blog-the-hilt-thereof-was-of-pure-gold/, accessed July 11, 2016.

103. S. Kent Brown, “What Were Those Sacrifices Offered by Lehi?” in From Jerusalem to Zarahemla: Literary and Historical Studies of the Book of Mormon (Provo, UT: Religious Studies Center, Brigham Young University, 1998), 1–8, https://rsc.byu.edu/jerusalem-zarahemla/what-were-those-sacrifices-offered-lehi.

104. John A. Tvedtnes, “Books in the Treasury,” The Book of Mormon and Other Hidden Books (Provo, UT: Foundation for Ancient Research and Mormon Studies, 2000) 155–166, https://scholarsarchive.byu.edu/cgi/viewcontent.cgi?filename=11&article=1081&context=mi&type=additional.

105. Exemplary sources include the following books: Warren P. Aston and Michaela K. Aston, In the Footsteps of Lehi (Salt Lake City, UT: Deseret Book Comp., 1994); Warren P. Aston, Lehi and Sariah in Arabia: The Old World Setting of the Book of Mormon (Bloomington, IN: Xlibris Publishing, 2015); and George Potter and Richard Wellington, Lehi in the Wilderness: 81 New, Documented Evidences that the Book of Mormon is a True History (Springville, UT: Cedar Fort, Inc., 2003). For videos, see Lehi in Arabia, DVD, directed by Chad Aston (Brisbane, Australia: Aston Productions, 2015) and Journey of Faith, DVD, directed by Peter Johnson (Provo, UT: Neal A. Maxwell Institute of Religious Scholarship, 2006), http://journeyoffaithfilms.com/. For a response to criticism of the abundant evidence from Lehi’s Trail, see Lindsay, “Joseph and the Amazing Technicolor Dream Map,” Parts 1 and 2.

106. See Neal Rappleye, “’Idle and Slothful Strange Stories’: Book of Mormon Origins and the Historical Record,” Interpreter: A Journal of Mormon Scripture 20 (2016): 21–37, https://journal.interpreterfoundation.org/idle-and-slothful-strange-stories-book-of-mormon-origins-and-the-historical-record/. For the basic case for historicity, see Stephen O. Smoot, “The Imperative for a Historical Book of Mormon,” The Interpreter Foundation Blog, Oct. 20, 2013, https://interpreterfoundation.org/blog-the-imperative-for-a-historical-book-of-mormon/.

107. See Christine Meilicke, “Moses’ Staff and the Return of the Dead,” Jewish Studies Quarterly, 6/4 (1999): 345–372, with the citation at 353, http://www.jstor.org/stable/40753246.

108. Ibid., 347.

109. Of course, if the Book of Mormon were a purely modern work from the mind of Joseph Smith, the more relevant question might be how rods were perceived in Joseph Smith’s environment, where divining rods of metal and sometimes even of steel were used to lead people to desired targets such as water or treasure. Regarding Joseph Smith’s involvement in such activities, see Mark Ashurst McGee, A Pathway to Prophethood: Joseph Smith Junior as Rodsman, Village Seer, and Judeo-Christian Prophet, Chapter 3, “Joseph Smith and The Gift of Working with the Rod,” (master’s thesis, Utah State University, 2000), 122–55; available for purchase from Proquest.com.

110. John A. Tvedtnes and David E. Bokovoy, “Rod as a Symbol of Power,” Testaments: Links Between the Book of Mormon and the Hebrew Bible (Tooele, UT: Heritage Distribution, 2003), 43–44.

111. Matthew Bowen, “What Meaneth the Rod of Iron?,” Insights 25/2 (2005): 2–3, http://publications.mi.byu.edu/fullscreen/?pub=1313&index=3.

112. “Walking stick (hieroglyph),” Wikipedia, https://commons.wikimedia.org/wiki/Category:Walking_stick_%28hieroglyph%29, accessed July 31, 2016.

113. Bowen, “What Meaneth the Rod of Iron?,” footnote 4.

114. John Tvedtnes, “Rod and Sword as the Word of God,” Journal of Book of Mormon Studies 5/2 (1996) 148–55, https://scholarsarchive.byu.edu/cgi/viewcontent.cgi?article=1124&context=jbms.

115. RT, “Critiques of Nahom and Lehi’s Journey.”

116. “Iron Age,” Wikipedia, https://en.wikipedia.org/wiki/Iron_Age, accessed July 14, 2016.

117. Matthew Roper, “Laban’s Sword of ‘Most Precious Steel’ (Howlers #5),” FAIRMormon Blog, June 17, 2013, http://blog.fairmormon.org/2013/06/17/labans-sword-of-most-precious-steel-howlers-5-2/.

118. It is often said that Psalm 2 came after the Exile. A discussion of evidence of a possibly more ancient origin is found in William H. Brownlee, “Psalms 1–2 as a Coronation Liturgy,” Biblica 52/3 (1971): 321–336, http://www.jstor.org/stable/42609595.

119. Strong’s H1280, BlueLetterBible.org, https://www.blueletterbible.org/lang/lexicon/lexicon.cfm?Strongs=H1280&t=kjv. See the section with Gesenius’ Lexicon for H1280.

120. Strong’s H982, BlueLetterBible.org, https://www.blueletterbible.org/lang/lexicon/lexicon.cfm?Strongs=H5982&t=kjv. See particularly the section with Gesenius’ Lexicon for H5982.

121. Margaret Barker, “Joseph Smith and Preexilic Israelite Religion,” in The Worlds of Joseph Smith: The Worlds of Joseph Smith: A Bicentennial Conference at the Library of Congress, ed. John S. Welch (Provo, UT: Brigham Young University Press: 2006), Kindle edition, section “White Fruit and a Guiding Rod.”

122. Some contend that the biblical themes it draws upon are impossibly late, such as drawing upon Revelations 22:1–2 for the concept of the water of life/river of water near the tree of life. However, John A. Tvedtnes shows that those concepts are grounded in the Old Testament and pose no special difficulty for the Book of Mormon. See John A. Tvedtnes, “1 Nephi 8:10, 13. Tree of Life,” Book of Mormon Research, Aug. 22, 2008, http://bookofmormonresearch.org/book-of-mormon-criticisms/specific-criticisms/criticisms-1-nephi/1-nephi-810-13-tree-of-life, accessed Aug. 3, 2016.

123. For discussions of ancient tree of life themes, see Simo Parpola, “The Assyrian Tree of Life: Tracing the Origins of Jewish Monotheism and Greek Philosophy,” Journal of Near Eastern Studies, 52/3 (July 1993): 161–208, http://www.jstor.org/stable/545436. The relationship between the ancient Mesopotamian tree and the concept of ascent into heaven is discussed at 167–173 and 195–197. Also see E. O. James, “The Tree of Life,” Folklore, 79/4 (Winter, 1968): 241–249, http://www.jstor.org/stable/1259355. Also see Zofja Ameisenowa and W. F. Mainland, “The Tree of Life in Jewish Iconography,” Journal of the Warburg Institute, 2/4 (April 1939): 326–345, http://www.jstor.org/stable/750042.

124. C. Wilfred Griggs, “The Tree of Life in Ancient Cultures,” Ensign 18 (June 1988): 26–31; https://churchofjesuschrist.org/ensign/1988/06/the-tree-of-life-in-ancient-cultures?lang=eng.

125. On the Origin of the World, in The Nag Hammadi Library in English, ed. James M. Robinson (New York: Harper and Row, 1977), 2.5.110, p. 169, as cited by Barker, “Joseph Smith and Preexilic Israelite Religion.” The relevant text can be viewed at Early Christian Writings, http://www.earlychristianwritings.com/text/originworld.html, or Gnosis.org, http://gnosis.org/naghamm/origin.html: “Now the color of the tree of life is like the sun. And its branches are beautiful. Its leaves are like those of the cypress. Its fruit is like a bunch of grapes when it is white.”

126. Here Barker cites Peterson, “Nephi and His Asherah.”

127. Barker, “Joseph Smith and Preexilic Israelite Religion.”

128. Peterson, “Nephi and His Asherah.” Also see Also see “Lady Wisdom,” chapter 9 in Murphy, The Tree of Life, 133–147.

129. Brant Gardner, Traditions of the Fathers: The Book of Mormon as History (Salt Lake City: Greg Kofford Books, 2015), 98.

130. Peterson, “Nephi and His Asherah,” 22–25.

131. John Tvedtnes, “Mary and the Tree of Life,” Book of Mormon Research, Nov. 8, 2008, http://bookofmormonresearch.org/index/book_of_mormon_articles/mary-and-the-tree-of-life.

132. D. John Butler’s book, Plain and Precious Things: The Temple Religion of the Book of Mormon’s Visionary Men (John D. Butler, 2012), available as an ebook at Smashwords.com (http://www.smashwords.com/books/view/137333) or as a paperback from Amazon.com.

133. Jeff Lindsay, “A Temple Gone Dark: An Important New Slant on the Themes of Nephi’s Vision and Lehi’s Dream,” Mormanity Blog, Sept. 20, 2012, http://mormanity.blogspot.hk/2012/09/a-temple-gone-dark-important-new-slant.html.

134. Butler, Plain and Precious Things, 12–13.

135. Ibid., 20–21.

136. Ibid., 22–23.

137. Ibid., 23.

138. Ibid., 25–26.

139. John S. Welch, “Straight (Not Strait) and Narrow,” Journal of Book of Mormon Studies 16/1 (2007): 18–25, 83–84, https://scholarsarchive.byu.edu/cgi/viewcontent.cgi?article=1415&context=jbms (text only) or https://scholarsarchive.byu.edu/cgi/viewcontent.cgi?article=1415&context=jbms (PDF).

140. John Bunyan, Pilgrim’s Progress (London: Nathaniel Ponder, 1678), http://www.gutenberg.org/files/39452/39452-h/39452-h.htm, accessed Aug. 4, 2016.

141. Welch, “Straight (Not Strait) and Narrow,” 19.

142. Cyprian Treatise 12.3.6, “Three Books of Testimonies against the Jews,” in The Ante-Nicene Fathers, vol. 5, ed. Alexander Roberts and James Donaldson (Grand Rapids: Eerdmans, 1965), 534, and Epistles of Cyprian 6.3 in Ante-Nicene Fathers, 5:284, both as cited by Welch, “Straight (Not Strait) and Narrow.”

143. Origen, Commentary on John 10.28, in Ante-Nicene Fathers, vol. 10, ed. Allan Menzies (Grand Rapids, MI: Eerdmans, 1969), 408, as cited by Welch, “Straight (Not Strait) and Narrow.”

144. “Early Modern English,” Wikipedia, https://en.wikipedia.org/wiki/Early_Modern_English, accessed July 28, 2016.

145. John Dee, Dr. Dee’s Apology (1591), https://books.google.com/books?id=iikdAQAAMAAJ&pg=PP64.

146. Richard Hooker, Of the Lawes of Ecclesiastical Politie (1632), 318, https://books.google.com/books?id=5AI8AQAAMAAJ&q=%22straight+and+narrow%22&dq=%22straight+and+narrow%22&hl=en&sa=X&ved=0ahUKEwjpqu25-obNAhVE2GMKHcDNA1AQ6AEIJTad.

147. Edward Chetwind, The Strait Gate and Narrow Way to Life, Opened and Pointed Out Upon Luk. 13, 23, 24 … The Second Edition, Corrected and Revised (London: John Beale, 1632), 165, https://books.google.com/books?id=D_FmAAAAcAAJ&pg=PA165&dq=%22strait+and+narrow%22&hl=en&sa=X&ved=0ahUKEwiDsN2–YbNAhUL_mMKHcBzBQgQ6AEIKDAC#v=onepage&q=%22strait%20and%20narrow%22&f=false.

148. Early English Books Online (EEBO), University of Michigan, http://quod.lib.umich.edu/cgi/t/text/text-idx?c=eebo;c=eebo2;g=eebogroup;xc=1;page=boolean.

149. Robert Albott, Englands Parnassus: or the choysest flowers of our moderne poets (London: N. Ling, C. Burby and T. Hayes, 1600.), http://quod.lib.umich.edu/e/eebo?ALLSELECTED=1;c=eebo;c=eebo2;didno=A16884.0001.001;g=eebogroup;rgn=works;singlegenre=All;size=25;sort=occur;start=1;subview=detail;type=boolean;view=reslist;xc=1;q1=straight+and+narrow.

150. Thomas Bell, The Iesuits antepast conteining, a repy against a pretensed aunswere to the Downe-fall of poperie, lately published by a masked Iesuite Robert Parsons by name, though he hide himselfe couertly vnder the letters of S.R. which may fitly be interpreted (a sawcy rebell.) (London: William Jaggard, 1608), http://quod.lib.umich.edu/e/eebo?ALLSELECTED=1;c=eebo;c=eebo2;didno=A07868.0001.001;g=eebogroup;rgn=full+text;singlegenre=All;size=25;sort=occur;start=1;subview=detail;type=simple;view=reslist;xc=1;q1=straight+and+narrow.

151. Stanford Carmack, “A Look at Some ‘Nonstandard’ Book of Mormon Grammar,” Interpreter: A Journal of Mormon Scripture 11 (2014): 209–262, https://journal.interpreterfoundation.org/a-look-at-some-nonstandard-book-of-mormon-grammar/; Stanford Carmack, “Why the Oxford English Dictionary (and not Webster’s 1828),” Interpreter: A Journal of Mormon Scripture 15 (2015): 65–77, https://journal.interpreterfoundation.org/why-the-oxford-english-dictionary-and-not-websters-1828/; and Stanford Carmack, “What Command Syntax Tells Us About Book of Mormon Authorship,” Interpreter: A Journal of Mormon Scripture 13 (2015): 175–217, https://journal.interpreterfoundation.org/what-command-syntax-tells-us-about-book-of-mormon-authorship/. Also see several related posts on the Mormanity Blog available at http://mormanity.blogspot.com/search?q=carmack+stanford.

152. Comment posted at Jeff Lindsay, “The Iron Rod: Inspired by an Aqueduct in Rochester?,” Mormanity Blog, May 30, 2016; http://mormanity.blogspot.com/2016/05/the-iron-rod-inspired-by-aqueduct-in.html.

153. Joseph’s editing of the Book of Mormon text after dictation and the initial printing points to both tight control during dictation but an overall willingness to accept a form of loose control since he was willing to edit some things from the dictation that he didn’t like. The edited elements include what appear to be Hebraisms or valid Early Modern English but awkward modern English. See, for example, Carmack, “A Look at Some ‘Nonstandard’ Book of Mormon Grammar,” and John Tvedtnes, “The Hebrew Background of the Book of Mormon,” Rediscovering the Book of Mormon, ed. John L. Sorenson and Melvin J. Thorne (Salt Lake City: Deseret Book and FARMS, 1991), 77–91, https://scholarsarchive.byu.edu/cgi/viewcontent.cgi?filename=8&article=1064&context=mi&type=additional.

154. “What was the Great and Terrible Gulf in Lehi’s Dream?,” Book of Mormon Central, Jan. 19, 2016, https://knowhy.bookofmormoncentral.org/content/what-was-great-and-terrible-gulf-lehi%E2%80%99s-dream, accessed June 28, 2015.

155. S. Kent Brown, “New Light From Arabia on Lehi’s Trail,” in Echoes and Evidences of the Book of Mormon, ed. Donald W. Parry, Daniel C. Peterson, and John W. Welch (Provo, Utah: FARMS, 2002), 64, https://scholarsarchive.byu.edu/cgi/viewcontent.cgi?filename=4&article=1026&context=mi&type=additional.

156. Ibid. Brown cites Jean-François Breton, “Architecture,” in Queen of Sheba: Treasures from Ancient Yemen, ed. St. John Simpson (London: The British Museum Press, 2002), 142–48, “Architecture,” with quotations from 143.

157. Yusuf ‘Abdullah, “The Past Lives on: Man, Landscape, and History in Yemen,” in Yemen: 3000 Years of Art and Civilisation in Arabia Felix, ed. Werner Daum (Innsbruck, Austria: Pinguin-Verlag, 1987), 472–479, quote at 472.

158. Ibid., 473.

159. Further, Brown’s discussion of the word “strange” from Lehi may miss another possible meaning of “strange” as in “strange flesh” in Jude 1:7, suggestive of a perversion rather than an unusual appearance, as one anonymous peer reviewer of this paper observed. “Similarly, a strange building could absolutely refer to a corrupted temple” (Personal communication, Sept. 29, 2016). Instead of referring to the architecture, Lehi may have been emphasizing the perversion of religion represented by the great and spacious building. The reviewer also observes that if Lehi’s dream were anticipating tall buildings they would later encounter, it seems odd that Nephi would not mention them when encountered later, though this theoretically could have been mentioned in the lost 116 pages.

160. Ibid.

161. Photo of the high-rise architectures at Shibam, Wadi Hadhramaut, from Jialiang Gao, available at “Shibam,” Wikipedia, https://en.wikipedia.org/wiki/Shibam, accessed Aug. 2, 2016, licensed under the Creative Commons Attribution-Share Alike 3.0 Unported license.

162. Grunder, “The Dream of the Iron Rod,” 1386.

163. Warren P. Aston, Lehi and Sariah in Arabia: The Old World Setting of the Book of Mormon (Bloomington, IN: Xlibris Publishing, 2015); also see Lindsay, “Joseph and the Amazing Technicolor Dream Map,” Parts 1 and 2.

164. Lindsay, “Joseph and the Amazing Technicolor Dream Map,” Part 1, at point #34 of the brief responses to RT.

165. Aston, Lehi and Sariah in Arabia.

166. Christine Meilicke observes that the staff of Moses “embodies the power to split, to separate and to divide. It is used in making a distinction between the Israelites and the enemies of God by bringing redemption to the Israelites and at the same time destroying their adversaries.” Meilicke, “Moses’ Staff and the Return of the Dead,” 352.

167. Tvedtnes, “Rod and Sword as the Word of God.” Regarding criticism of Nephi’s “fiery darts” in this passage, see Stephen O. Smoot, “The ‘Fiery Darts of the Adversary’ in 1 Nephi 15:24,” Interpreter: A Journal of Mormon Scripture 18 (2016): 5–9, https://journal.interpreterfoundation.org/the-fiery-darts-of-the-adversary-in-1-nephi-1524/.

168. Zachary Nelson, “The Rod of Iron in Lehi’s Dream,” Religious Educator 10/3 (2009): 49–58, https://rsc.byu.edu/archived/volume-10-number-3-2009/rod-iron-lehi-s-dream.

169. This theme of grasping and not letting go in order to receive a blessing may also relate to the tale of Jacob wrestling the angel in Genesis 32:22–31. See also Philippians 3:12–14; 1 Thessalonians 5:21; 1 Timothy 6:12; Hebrews 3:6, etc.

170. Royal Skousen, Analysis of Textual Variants of the Book of Mormon, Part One: 1 Nephi 1 – 2 Nephi 10 (Provo, UT: Foundation for Ancient Research and Mormon Studies, 2014), 257–258, https://interpreterfoundation.org/books/atv/p1/. Also see Royal Skousen, The Book of Mormon: The Earliest Text (New Haven, CT and London: Yale University Press, 2009), 32, 748 (1 Nephi 12:18).

171. Tvedtnes, “Rod and Sword as the Word of God.”

172. Ibid.

173. Joshua Berman, “The ‘Sword of Mouths’ (Judges III. 16; Psalms CXLIX 6; Proverbs V 4): A Metaphor and Its Ancient Near Eastern Context,” Vetus Testamentum, 52/3 (July 2002): 291–303, http://www.jstor.org/stable/1585054.

174. Ibid., 292–93.

175. Yuval Harari, “Moses, the Sword, and The Sword of Moses: Between Rabbinical and Magical traditions,” Jewish Studies Quarterly, 12/4 (2005): 293–329, with citations at 303, 318 and 325, http://www.jstor.org/stable/40753384.

176. Strong’s H1870, Blue Letter Bible, https://www.blueletterbible.org/lang/Lexicon/Lexicon.cfm?strongs=H1870&t=kjv.